You are on page 1of 465

‫صدقة جارية لي و الهلي‪ ,‬أرجوكم ال تنسونا من‬

‫صالح دعائكم‬
‫‪Please Grace us with your good prayers‬‬

‫يوسف معيوف‬
‫‪Youssef Maayouf‬‬
Can you sum drug metabolism in Two lines?

•phase I: oxidation, reduction, hydrolysis

•phase II: conjugation


Can you describe the phase 1 of drug
metabolism?
• Phase I reactions: oxidation, reduction, and hydrolysis.
• Mainly performed by the P450 enzymes but some drugs are
metabolised by specific enzymes, for example alcohol dehydrogenase
and xanthine oxidase. Products of phase I reactions are typically
more active and potentially toxic
Can you describe the phase 2 of drug
metabolism?
• Phase II reactions: conjugation.
• Products are typically inactive and excreted in urine or bile.
• Glucuronyl, acetyl, methyl, sulphate and other groups are typically
involved.

• The majority of phase I and phase II reactions take place in the liver.
What is first pass metabolism?

• First-Pass Metabolism is a phenomenon where the concentration of a


drug is greatly decreased before it reaches the systemic circulation
due to hepatic metabolism.

• As a consequence much larger doses are need orally than if given by


other routes.
What are the drugs that will suffer from first
pass metabolism?
• (Angina patient that had a VT so took Lignocaine)
• Aspirin
• Isosorbide dinitrate
• Glyceryl trinitrate
• Propranolol
• Verapamil
• Lignocaine
What are Zero order kinetics?

• Zero-Order Kinetics describes metabolism which is independent of the


concentration of the reactant.
• This is due to metabolic pathways becoming saturated resulting in a constant
amount of drug being eliminated per unit time.

• This explains why people may fail a breathalyser test in the morning if they
have been drinking the night before
What are the drugs that suffer from Zero
order kinetics?
• (PESH is rekhem, he has epilepsy thrombus and drinks)

• Phenytoin
• Salicylates
• Heparin
• Ethanol
What is the Acetylator status in UK?

• Acetylator Status
• 50% of the UK population is deficient in hepatic N-acetyltransferase
What are the drugs that are affected by the
acetylator status (hepatic N-
acetyltransferase)?
• (Acetylator IS PHD)
• Isoniazid
• Sulfasalazine
• Procainamide
• Hydralazine
• Dapsone
What are the P450 (Phase 1 of metabolism)
dependent drugs?

• P-450 Dependent Drugs WEPTD:

• Warfarin
• Estrogen
• Phenytoin
• Theophylline

• Digoxin
What are the drugs that inhibit P450?

• Verapamil
• Acute alcohol intake
• Allopurinol
• Amiodarone
• Cimetidine, omeprazole
• Dapsone
• Imidazoles: ketoconazole, fluconazole
• INH
• Macrolides (Azithro-Clarithro-Erythro mycins)
• Quinolones (ciprofloxacin)
• Quinupristin
• Sodium valproate
What are the drugs that inhibit P450?

• Spironolactones

• SSRIs: fluoxetine, sertraline

• Grapefruit juice (potent inhibitor of the cytochrome P450 enzyme CYP3A4)

• Protease inhibitors (ndinavir, nelfinavir, ritonavir, saquinavir)


Ciclosporin is affected by Enzyme inducers
and inhibitors, T/F?
• True
Carbamazepine is an inducer of the P450
system. This in turn increases the metabolism of
carbamazepine itself – autoinduction, T/F?

• True
What are the drugs that induces P450?

• Antiepileptics: phenytoin, carbamazepine (note that valporate is an inhibitor)


• Barbiturates

• Chronic alcohol intake

• Griseofulvin
• Quinidine

• Rifampicin
What are the drugs that induces P450?

• Smoking (affects CYP1A2, reason why smokers require more aminophylline)

• St John's Wort

• Sulfa drugs

• Tetracycline

• Nevirapine (NNRTI)
Which drugs can be cleared with
Hemodialysis (can use hemodialysis in case
of toxicity)?
• Mnemonic BLAST
• Barbiturate
• Lithium

• Alcohol (inc methanol, ethylene glycol)

• Salicylates

• Theophyllines (charcoal hemoperfusion is preferable)


Which drugs cannot be cleared with Hemodialysis
(you can’t use hemodialysis in case of toxicity)?

• Tricyclics

• Benzodiazepines (diazepam,midazolam,alprazolam)

• Dextropropoxyphene (co-proxamol)

• Digoxin, Beta-blockers
Which drugs should you avoid in Renal
failure?
• Antibiotics: tetracycline, nitrofurantoin

• NSAIDS

• Lithium
Which Drugs are likely to accumulate in renal
failure (need dose adjustment)?
• Most antibiotics including penicillins, cephalosporins, vancomycin, gentamicin,
streptomycin

• Digoxin, atenolol

• Methotrexate

• Sulphonylureas
• Furosemide
• Opioids
Which Drugs relatively safe in renal failure
(you can use in normal dose)

•Antibiotics: erythromycin, rifampicin

•Diazepam

•Warfarin
What are the drugs that you can’t take in
Renal failure?
Many People Like To Nap Naked
•Metformin
•Potassium Sparing Diuretics
•Lithium
•Tetracycline
•Nitrofurantoin
•NSAID (Caution
Which Drugs relatively safe in renal failure
(you can use in normal dose)
WERD
•Antibiotics: erythromycin, rifampicin

•Diazepam

•Warfarin
Which drugs causes imapired glucose
tolerance?
• Thiazides, furosemide (less common)

• Steroids

• Tacrolimus, cyclosporin
• Interferon- alpha
• Nicotinic acid (vitamin B3)
Beta-blockers cause a slight impairment of glucose tolerance.
They should also be used with caution in diabetics as they can
interfere with the metabolic and autonomic responses to
hypoglycemia, T/F?
• True
How can we classify Drug induced liver
disease?
• Drug induced Liver Disease is generally divided into hepatocellular,
cholestatic or mixed.

• There is however considerable overlap, with some drugs causing a


range of changes to the liver
What are the drugs that causes hebatocellular drug
induced liver disease?
• Alcohol
• Amiodarone
• Anti-tuberculosis: isoniazid,
• rifampicin, pyrazinamide
• Halothane
• MAOIs
• Methyldopa
• Paracetamol
• Sodium valproate, phenytoin
• Statins
What are the drugs that causes Cholestasis +/-
Hebatitis drug induced liver disease?
• Anabolic steroids, testosterones
• Antibiotics: flucloxacillin, co-amoxiclav, erythromycin*, nitrofurantoin
• Fibrates
• Oral contraceptive pill
• Phenothiazines: chlorpromazine,
• prochlorperazine
• Rarely: nifedipine
• Sulphonylureas
Cholestatic Drugs:
• CHlorpromazine
• Ofloxacin
• Largactil (ie chlorpromazine...cldn't think of any L-drugs)
• Erythromycin
• Sulfamethoxazole
• +
• Trimethoprim (Co-Trimox)
• Augmentin (ie Co-Amoxiclav)
• Tetracycline
• Ibuprofen
• Cimetidine
The risk of drug induced liver disease may be
decreased with erythromycin stearate than
Erythromycin, T/F?
• True
Which drugs might induce liver cirrhosis?

• MAM
• Methotrexate
• Amiodarone
• Methyldopa
Which drugs might cause Cataracts?

•Steroids
Which drugs might cause Corneal opacities?

• Amiodarone

• Indomethacin
Which drugs might cause Optic neuritis?

• Ethambutol

• Amiodarone

• Metronidazole
Which drugs might cause Retinopathy?

•Chloroquine, quinine
Which drugs might cause Blue tinge in
vision?

•Sildinafil (The Blue Pill causes blue vision)


Which drugs might cause Yellow-green
tinge in vision?

•Digoxin
Sildenafil can cause both blue discoloration
and non-arteritic anterior ischemic
neuropathy, T/F?
• True
What are the Drugs Causing Gingival
hyperplasia?
• Phenytoin

• Cyclosporin

• Calcium channel blockers (especially nifedipine)


Which disease causes Gingival Hyperplasia?

•Acute myeloid leukemia (myelomonocytic and monocytic types)


What are the drugs that might cause
Urticaria?
• The following drugs commonly cause urticaria:
OPAN

• Opiates
• Penicillins
• Aspirin
• NSAIDs
Which drugs might precipitate an attack of
porphyria?
• Alcohol

• Barbiturates
• Benzodiazepines

• Halothane

• Oral contraceptive pill

• Sulphonamides
How do we diagnose acute intermittent
porphyria ?

• Diagnosis is by Ehrlich’s test: add one volume of Ehrlich reagent to


one volume of urine, the sample turns red.
• Add two volumes of chloroform and the red colour stays in the upper
(aqueous) layer.
• Urinary delta aminolevulonic acid and porphobilinogen are both
raised.
Which drugs will you use for acute
intermittent porphyria?
• Haematin can help patients as it acts as a negative feedback operator
on the porphyrin synthetic pathway.
• High doses of glucose (400 g/day) can inhibit heme synthesis and are
useful for treatment of mild attacks. The patient should receive a
high-carbohydrate diet during an acute episode.
• As a general rule enzyme inducers exacerbate porphyria as they
induce the pathway.
Which drugs might precipitate an attack of
porphyria?
• As a general rule enzyme inducers exacerbate porphyria as they induce the
pathway.
• Alcohol STOP RBC
S- SULFONAMIDE
T-THIOPENTONE
O-OC PILLS
P-Benzo
R-RIFAMPICIN
B-BARBI
C-CHLORMPHNICOL
Which drugs are safe with Porphyria?

• Paracetamol
• Aspirin
• Codeine
• Morphine
• Chlorpromazine
• Beta -blockers
• Penicillin
• Metformin
Which drugs may induce Thrombocytopenia?

• (probable immune mediated)


• HAND Q Anti Anti
• Heparin
• Abciximab
• NSAIDs; ASA
• Diuretics: furosemide
• Quinine
• Antibiotics: penicillins, sulphonamides, rifampicin
• Anticonvulsants: carbamazepine, valproate
Which drugs might cause drug induced
pancytopenia?
• Cytotoxics
• Antibiotics: trimethoprim, chloramphenicol
• Anti-rheumatoid: gold (sodium aurothiomalate), penicillamine

• Carbimazole(causes both agranulocytosis and pancytopenia)

• Anti-epileptics: carbamazepine
• Sulphonylureas: tolbutamide
Carbimazole causes both agranulocytosis and
pancytopenia, T/F?

• True
Rash on the forearms and face is typical of a
photosensitivity rash, T/F?
• True
Which drugs might induce drug induced
photosensetivity rash?
• PQRST
• Phenothiazides, Psoralens
• Quinines
• Retinoids
• Sulphonylureas, sulphonamides
• Tetracyclines,, ciprofloxacin

• Amiodarone
• NSAIDs e.g. Piroxicam
What are the NICE2008 guidelines for
smoking cessation therapy?
• Patients should be offered nicotine replacement therapy (NRT), varenicline or
bupropion -NICE state that clinicians should not favour one medication over
another

• NRT, varenicline or bupropion should normally be prescribed as part of a


commitment to stop smoking on or before a particular date (target stop date)

• Do not offer NRT, varenicline or bupropion in any combination


Can smoking cessation therapy be offered in
combination?
• No
• Do not offer NRT, varenicline or bupropion in any combination
What are the guidelines for prescribing NRT,
Varenicline, Bupropion for smoking
cessation therapy?
• Prescription of NRT, varenicline or bupropion should be sufficient to last only until
2 weeks after the target stop date.
• Normally, this will be after 2 weeks of NRT therapy, and 3-4 weeks for varenicline
and bupropion, to allow for the different methods of administration and mode of
action. Further prescriptions should be given only to people who have
demonstrated that their quit attempt is continuing

• If unsuccessful using NRT, varenicline or bupropion, do not offer a repeat


prescription within 6 months unless special circumstances have intervened
What do you know about Nicotine
Replacement Therapy for smoking cessation
therapy?
• Adverse effects include nausea & vomiting, headaches and flu-like symptoms

• Nice recommend offering a combination of nicotine patches and another form of


NRT (such as gum, inhalator, lozenge or nasal spray) to people who show a high
level of dependence on nicotine or who have found single forms of NRT
inadequate in the past
What do you know about Varenicline for
smoking cessation therapy?
• Nicotinic receptor partial agonist
• Suicide Vomiting is caused by Varnecline which is Nicotine
• Should be started 1 week before the patient target date to stop
• The recommended course of is 12 weeks (but patients should be monitored regularly and
treatment only continued if not smoking)
• Has been shown in studies to be more effective than bupropion
• Nausea is the most common adverse effect. Other include headache, insomnia, abnormal dreams
• Varenicline should be used with caution in patients with a history of depression or self-harm.
• Contraindicated in pregnancy and breast feeding
What do you know about Bupropion for
smoking cessation therapy?
• Should be started 1 to 2 weeks before target date.
• Small risk of seizures (1: 1,000)
• Bupropion should not be prescribed to individuals with epilepsy or other
conditions that lower the seizure threshold, such as alcohol or
benzodiazepine withdrawal, anorexia nervosa, bulimia, or active brain
tumors.
• It should be avoided in individuals who are also taking MAOIs. When
switching from MAOIs to bupropion, it is important to include a washout
period of 2 weeks.
• Also pregnancy and breastfeeding are contraindications.
What is the relationship between salicylate
toxicity and Acidosis and alkalosis?
• a key concept for the exam is to understand that salicylate overdose leads to a
mixed respiratory alkalosis and metabolic acidosis.
• Early stimulation of the respiratory centre leads to a respiratory alkalosis whilst
later the direct acid effects of salicylates (combined with acute renal failure) may
lead to an acidosis.

• In children metabolic acidosis tends to predominate


When would you suspect a patient with
Salicylate toxicity?
• The mixed respiratory alkalosis and metabolic acidosis in a sweaty,
confused patient point towards salicylate overdose.

• The development of pulmonary edema suggests severe poisoning


and is an indication for hemodialysis
What are the features of Salicylate toxicity?

• DAFT HID
• Deafness. Appear flushed. Fever. Tinnitus.
• Hyperventilation. Hyperglycemia and hypoglycemia
• Increased sweating,
• Dizziness

• Seizures
• Coma
What is the treatment of Salicylate toxicity?

• General (ABC, charcoal) Activated charcoal should be given even 6-8 hours after
overdose

• Urinary alkalinization is now rarely used - it is contraindicated in cerebral and


pulmonary edema with most units now proceeding straight to hemodialysis in
cases of severe poisoning
• Potassium depletion should be corrected with potassium supplementation into any
intravenous fluid replacement.
• Hemodialysis
What are the indications for Hemodialysis in
salicylate overdose?
• Serum concentration > 700mg/L
• Metabolic acidosis resistant to treatment

• Acute renal failure


• Pulmonary edema
• Seizures
• Coma
What is the biological effect of salicylate?

• salicylates cause the uncoupling of oxidative phosphorylation leading


to decreased (ADP) adenosine triphosphate production, increased
oxygen consumption and increased carbon dioxide and heat
production
What is the management of Paracetamol
overdose?
• Start N-acetyl cysteine immediately

• Naloxone if there is hypoxia or respiratory depression


What can you tell me about Renal failure
associated with paracetamol overdose?
• Acute tubular necrosis, Renal failure associated with paracetamol overdose is
recoverable and there is a reasonable chance of attaining normal renal
function after the acute event.
• Patients should be put on Haemodyalisis till they recover
When should you go for the liver
transplantation in case of paracetamol
toxicity?
• King's College Hospital criteria for liver transplantation (paracetamol liver failure)
Arterial pH < 7.3, 24 hours after ingestion OR all of the following:

• Prothrombin time > 100 seconds

• Creatinine > 300 μmol/l

• Grade III or IV encephalopathy


Why low pH is a bad prognostic indicator for
paracetamol toxicity?

• It indicates liver damage


• A pH below 7.3 is associated with only 15% chance of survival due to
failure of clearance of lactate by the liver.
What is the mechanism of Paracetamol
toxicity?
• When paracetamol is taken in large quantities, a minor metabolite called N-acetyl-p-
benzoquinone imine (NAPQI) builds up.

• It is normally conjugated by glutathione, but when taken in excess, the body's glutathione
reserves are not sufficient to inactivate the toxic NAPQI.
• This metabolite is then free to react with key hepatic enzymes P450, therefore damaging
hepatocytes.
• For this indication, acetylcysteine acts to augment the glutathione reserves in the body and,
together with glutathione, directly bind to toxic metabolites. These actions serve to protect
hepatocytes in the liver from NAPQI.
Which patients are at risk of developing
paracetamol toxicity after paracetamol
overdose?
• Chronic alcohol excess
• Patients on p450 enzyme inducers (rifampicin, phenytoin, carbamazepine)

• anorexia nervosa: decreased glutathione stores

• HIV
why there is a lower threshold for treating patients who
take P450 inducing medications e.g. phenytoin or
rifampicin when they take paracetamol overdose?

• Because the NAPQI is acted upon by the P450 enzyme which causes
the damage of the hebatic cells
What is Digoxin?

• Digoxin is a cardiac glycoside now used mainly in the management of


AF.
How long does digoxin need to start giving
the desired effect?
• The half-life of digoxin is around 36-48 hours.

• This results in a delay before steady plasma levels are seen, it may
take a week to start its action
What are the actions of Digoxin?

• Decrease conduction through the atrioventricular node which slows the


ventricular rate in atrial fibrillation and flutter

• Increase the force of cardiac muscle contraction due to inhibition of the Na+/K+
ATPase pump
What is the mechanism of actions of
Digoxin?
• Digoxin’s primary mechanism of action involves inhibition of the Na+/K+ ATPase, mainly in the
myocardium.
• This inhibition causes an increase in intracellular sodium levels, resulting in a reversal of the
action of the sodium-calcium exchanger, which normally imports three extracellular sodium ions
into the cell and transports one intracellular calcium ion out of the cell.
• The reversal of this exchange causes an increase in the intracellular calcium concentration that is
available to the contractile proteins.
• Increased intracellular calcium lengthens phase 4 and phase 0 of the cardiac action potential,
which leads to a decrease in heart rate.[19] Increased amounts of Ca2+ also leads to increased
storage of calcium in the sarcoplasmic reticulum, causing a corresponding increase in the release
of calcium during each action potential. This leads to increased contractility (the force of
contraction) of the heart without increasing heart energy expenditure.
What are the features of digoxin toxicity?

• Generally unwell, lethargy, nausea & vomiting, confusion, YELLOW-


GREEN vision

• Arrhythmias (e.g. AV block, bradycardia)


What are the precepitating factors for digoxin
toxicity?
• Classically: Hypokalemia
• Myocardial ischemia
• Hypomagnesemia, acidosis (Hypo pH), Hypercalcemia, Hypernatremia
• Hypoalbuminemia
• Hypothermia
• Hypothyroidism
• Drugs: amiodarone, quinidine, verapamil, spironolactone, Nifedipine (compete
for secretion in distal convoluted tubule therefore excretion)
Why spironolactone can cause digoxin
toxicity?
• spironolactone (compete for secretion in distal convoluted tubule
therefore decrease excretion)
What is the management of digoxin toxicity?

• Digibind

• Correct arrhythmias

• Monitor K+
What are the indications for giving Digoxin
specific antibodies (Digibind) in case of
digoxin toxicity?
• Severe hyperkalaemia (more than 6 mmol/L) resistant to treatment with insulin
and dextrose (not calcium gluconate risk of further ventricular arrythmias)
• Bradyarrythmia unresponsive to atropine with cardiac compromise
• Tachyarrythmia (especially ventricular) associated with cardiac compromise.
• Digoxin specific antibodies should be considered at an earlier stage if the patient
has pre-existing cardiac disease.
What are the Indications for administration of Digoxin
specific Fab Fragment when digoxin overdose?

• Hemodynamic instability
• Life-threatening arrhythmias

• Serum potassium >5 mmol/l in acute toxicity


• Plasma digoxin level >13nmol/l

• Ingestion of more than 10 mg digoxin in adults and 4 mg in children


What is the commercial use of cyanide?

• Cyanide may be used in insecticides, photograph development and the


production of certain metals.

• Toxicity results from reversible inhibition of cellular oxidising enzymes


Ap patient that was in a fire and developed
cyanide toxicity, why?

• Cyanide can be produced in house fires, when materials such as polyurethane


and rubber combust.
What is the presentation of Cyanide toxicity?

• Cyanide from Cyanosis


• Cyanide acts by irreversibly blocking mitochondrial electron transport
• 'Classical' features: BRICK-RED SKIN, smell of bitter almonds

• Acute: hypoxia, hypotension, headache, confusion

• Chronic: ataxia, peripheral neuropathy, dermatitis


What is the Management of Cyanide toxicity?

• Supportive measures: 100% oxygen

• Definitive: IV dicobalt edetate


• sodium nitrite or sodium thiosulphate may be given as alternatives.
• Mouth to mouth resuscitation should not be attempted where there
is a suspicion of cyanide exposure, due to risks to the resuscitator
What is Ethylene glycol?

• Ethylene glycol is a type of alcohol used as a COOLANT OR


ANTIFREEZE
What is the first line of treating toxicity of
Ethylene glycol?
• Fomepizole is now used first-line rather than ethanol in ethylene glycol toxicity.

• There is no indication for hemodialysis unless metabolic acidosis occurred or


severe toxicity (>8 mmol/l) or the case is refractory to antidotes
• For acidosis I can be corrected using sodium bicarbonate, given via a central line
What are the features of toxicity of Ethylene
glycol?
• Features of toxicity are divided into 3 stages:
• Stage 1: symptoms similar to alcohol intoxication: confusion, slurred speech,
dizziness
• Stage 2: metabolic acidosis with high anion gap and high osmolar gap. Also
tachycardia, hypertension

• Stage 3: acute renal failure


What is the mechanism of multi organ failure
in case of toxicity of Ethylene glycol?
• Multi-organ failure is thought to occur at least in part due to widespread
deposition of calcium oxalate crystals around 12hrs after the initial insult.
• High dose thiamine and pyridoxine should also be used as this reduces the
formation of oxalate from glyxoylic acid.
• Because of the possible formation of calcium oxalate, calcium levels should
also be assessed.
How can you differentiate between Ethylene
glycol and Methanol poisoning?

• The disc changes and normal calcium point towards methanol


over ethylene glycol as a causative agent and this could be confirmed
with methanol levels.
What is the management of toxicity of
Ethylene glycol?
• has changed recently:
• Ethanol has been used for many years
• Works by competing with ethylene glycol for the enzyme alcohol dehydrogenase,
this limits the formation of toxic metabolites (e.g. Glycoaldehyde and glycolic
acid) which are responsible for the hemodynamic/metabolic features of poisoning
• fomepizole, an inhibitor of alcohol dehydrogenase, is now used first-line in
preference to ethanol
• Hemodialysis also has a role in refractory cases
What are the main 3 areas of adverse effects
caused by cocaine?
• Cardiovascular effect

• Neurological effect

• Psychiatric effect
What are the cardiovascular adverse effects
caused by cocaine?
• Myocardial infarction
• Both tachycardia and bradycardia may occur

• Hypertension

• QRS widening and QT prolongation


• Aortic dissection
What are the neurological adverse effects
caused by cocaine?
• Seizures

• Hypertonia

• Hyperreflexia
What are the psychiatric adverse effects
caused by cocaine?
• Agitation

• Psychosis

• Hallucinations
What are the metabolic and muscular effects
of Cocaine?
• Hyperthermia

• Metabolic acidosis

• Rhabdomyolysis leading to renal failure.


What is the management of Cardiac
manifestations resulting from cocaine toxicity?

• Pretty much any complication of cocaine use is initially treated with


benzodiazepines.

In case of persistent palpitations after benzo, you need to give antiarrythmics


Beta-blockers should be avoided in cocaine toxicity due to the risk of coronary
vasospasm (see below). The most appropriate treatment for further palpitations
is verapamil.
What is Ecstasy?

• Ecstasy (MDMA, 3,4-Methylenedioxymethamphetamine) use became


popular in the 1990's during the emergence of dance music culture
What is the effect of Ecstasy usage?

• Neurological: agitation, anxiety, confusion, ataxia

• Cardiovascular: tachycardia, hypertension

• Water intoxication
• Hyperthermia
• Rhabdomyolysis
• Hyponatremia
How do we manage the ecstasy toxication?

• Supportive:
• Rapid convection cooling, in an attempt to reduce core temperature to below
38.3oC.
• Agitation is managed with use of benzodiazepines,
• IV rehydration is used to manage rhabdomyolysis, and IV agents such as
nitroprusside, which can be easily titrated, are optimal in the management of
hypertension
• Dantrolene may be used for hyperthermia if simple measures fail
What are the poor prognostic signs of ecstasy
toxication?
• Poor prognostic indicators in ecstasy overdose include hyperpyrexia
(>42oC), rhabdomyolysis, renal failure and liver failure.
What are the features of Mercury poisoning?

• Paraesthesia
• Visual field defects

• Hearing loss
• Irritability

• Renal tubular acidosis


What is the management of Mercury
poisoning?
• Chelation therapy for acute inorganic mercury poisoning can be done with DMSA,
2,3-dimercapto-1- propanesulfonic acid (DMPS), Dpenicillamine (DPCN), or
dimercaprol (BAL).

• Only DMSA is FDA-approved for use in children for treating mercury poisoning.
DMSA for treating mercury poisoning, yes or
no?
• However, several studies found no clear clinical benefit from DMSA treatment
for poisoning due to mercury vapor.

• No chelator for methylmercury or ethylmercury is approved by the FDA; DMSA is


the most frequently used for severe methylmercury poisoning, as it is given
orally, has fewer side effects, and has been found to be superior to BAL, DPCN,
and DMPS
What should you consider when having a question
of abdominal pain and neurological signs?

• Lead Poisoning: Along with acute intermittent porphyria, lead poisoning should
be considered in questions giving a combination of abdominal pain and
neurological signs
What are the features of Lead poisoning?

• Abdominal pain
• Peripheral neuropathy (mainly motor)

• Fatigue
• Constipation

• Blue lines on gum margin (only 20% of adult patients, very rare in children)
What is the effect of lead poisoning on
Kidney function?
• Hyperkalaemia occurs as a result of aldosterone resistance, also
known as renal tubular acidosis Type 4, because of lead
related renal damage.
• It also leads to chronic renal failure with mild to moderate
proteinuria
What are the features that you will see in the
investigations for Lead poisoning?
• Microcytic anemia
• Blood film shows red cell abnormalities including basophilic stippling and
cloverleaf morphology
• Raised serum and urine levels of delta aminolaevulinic acid may be seen making
it sometimes difficult to differentiate from acute intermittent porphyria

• Urinary coproporphyrin is also elevated (urinary porphobilinogen and


uroporphyrin levels are normal to slightly elevated)
Which tests would you do for investigations
for Lead poisoning?
• Since lead is concentrated in red cells, whole blood levels of lead rather than
plasma must be analysed, however testing for enzymes affected by lead is more
reliable than whole lead levels alone, and hence erythrocyte zinc protoporphyrin
(ZPP) levels and delta aminolevulinic acid dehydratase (ALAD) are more reliable. 
Why does microcytic anemia occur in Lead
poisoning?
• Because lead inhibits the conversion of delta-aminolevulinic acid dehydrase and
ferrochelatase, delta-aminolevulinic acid cannot be converted to
porphobilinogen, and iron is not incorporated into protoporphyrin.
• Hence, erythrocyte protoporphyrin is elevated to levels greater than 35mcg/dL,
a finding that is also common in iron deficiency anemia.
What is the management of Lead Poisoning?

• various chelating agents are currently used:


• Dimercaptosuccinic acid (DMSA)

• D-penicillamine
• EDTA (EthyleneDiamineTetraAcetic acid)

• Dimercaprol
What is the clinical significance of carbon
monoxide?
• Carbon Monoxide has high affinity for hemoglobin and myoglobin resulting in a
left-shift of the oxygen dissociation curve and tissue hypoxia.

• There are approximately 50 per year deaths from accidental carbon monoxide
poisoning in the UK.
Which items would be in a question about
carbon monoxide poisoning?
• Questions may hint at badly maintained housing e.g. student houses

• Confusion, pyrexia and pink mucosae are typical features of carbon monoxide
poisoning
What are the features of carbon monoxide
poisoning?
• Headache: 90% of cases
• Nausea and vomiting: 50%
• Vertigo: 50%
• Confusion: 30%
• Subjective weakness: 20%

• Severe toxicity: 'pink' skin and mucosae, hyperpyrexia, arrhythmias,


extrapyramidal features, coma, death
What are the different levels of Carbon
monoxide in the body and their significance?
• < 3% non-smokers
• < 10% smokers

• 10 - 30% symptomatic: headache, vomiting, dizziness


• > 30% severe toxicity:
• 50-60%: Syncope, tachycardia, fits
• > 60%: risk of cardiorespiratory failure and death
What is the management of Carbon
Monoxide toxicity?
• 100% oxygen

• Hyperbaric oxygen
What are the indications of Hyperbaric
oxygen when treating carbon monoxide
poisoning?
• Loss of consciousness at any point

• Neurological signs other than headache

• Myocardial ischemia or arrhythmia


• Pregnancy
What is Oculogyric Crisis?

• Oculogyric Crisis is a dystonic reaction to certain drugs or medical


conditions
What are the features of Oculogyric crisis?

• Features (extra pyramidal)

• Restlessness, agitation

• Involuntary upward deviation of the eyes


What are the causes of Oculogyric crisis?

• (All have antidopamenergic features)

• Phenothiazines
• Haloperidol
• Metoclopramide

• Postencephalitic Parkinson’s disease.


What is the pathophysiology of Oculogyric
crisis?
• Acute dystonic reactions are extrapyramidal side effects due to alteration of the
dopaminergic–cholinergic balance in the nigrostriatum.
• Medication-induced acute dystonia can be a side effect of treatment with
antiemetics, antipsychotics, antidepressants, antiepileptics, antimalarials, and
other drugs.
• Such drugs produce acute dystonic reactions through a nigrostriatal dopamine D2
receptor blockade, which results in an excess of striatal cholinergic output.
What is the treatment of oculogyric crisis?

• Procyclidine (anticholinergic drug ;) )

• Benztropine
Procyclidine is used in treatment of both
oculgyric crisis and Parkinsons specially drug
induced why?
• Because both have features of extrapyramidal side effects due to
alteration of the dopaminergic–cholinergic balance in the
nigrostriatum
• And procyclidine is anticholinergic
Which drugs are contraindicated in
pregnancy?
• ACE inhibitors, ARBs
• Statins
• Warfarin

• Sulfonylureas
• Retinoids (including topical)
• Cytotoxic agents
Which antibiotics are contraindicated in
pregnancy?
• MCAT
• Metronidazole (Some debate around it)
• Cloramphenicol
• Aminoglycosides
• Tetracyclines
• Trimethoprim
• Sulphonamides
• Quinolones: the BNF advises to avoid due to arthropathy in some animal studies
Should you stop anti-epileptic drugs during
pregnancy?
• Majority of antiepileptics including valproate, carbamazepine and
phenytoin are known to be potentially harmful.

• Decision to stop such treatments however is difficult as uncontrolled


epilepsy is also a risk
A mother should not breastfeed if she’s taking
antiepileptic drugs, T/F?

Breast feeding is acceptable with nearly all anti-epileptic drugs taken


in normal doses, with the possible exception of barbiturates
Which medical conditions are a
contraindication to breastfeeding?
• Galactosemia of the baby

• Viral infections - this is controversial with respect to HIV in the developing world.

• This is because there is such an increased infant mortality and morbidity


associated with bottle feeding that some doctors think the benefits outweigh the
risk of HIV transmission
Which drugs are safe with breastfeeding?

• Antibiotics: penicillins, cephalosporins, trimethoprim


• Endocrine: glucocorticoids (avoid high doses), levothyroxine*
• Epilepsy: sodium valproate, carbamazepine
• Asthma: salbutamol, theophyllines
• Psychiatric drugs: tricyclic antidepressants, antipsychotics (clozapine should be avoided)
• Hypertension: Beta-blockers, hydralazine, methyldopa
• Anticoagulants: warfarin, heparin
• Digoxin
Why Levothyroxine is safe to take with
breast feeding?
• BNF advises that the amount is too small to affect neonatal
hypothyroidism screening
Which drugs are dangerous with
breastfeeding?
• Antibiotics: ciprofloxacin, tetracycline,
• chloramphenicol, sulphonamides
• Psychiatric drugs: lithium, benzodiazepines, clozapine
• Aspirin
• Carbimazole
• Sulphonylureas
• Cytotoxic drugs
• Amiodarone
What’s the mnemonic for Drugs
contraindicated for breast feeding?
•  ALI CAN C BREAST (X2) "
Atropine
Lithium
Iodide

Cyclosporin
Alcohol
Nicotine

Chloramphenicol

Bromocryptine, Benzodiazipine
Rizatriptan, Radiopharmaceuticals
Ergotamine, Ethosuximide
Amiodarone, Amphetamines
Sex hormones, Stimulant laxatives
Tretinoin, Tetracycline
What are the different types of Heparin?

• Heparin can be given as either unfractionated, intravenous heparin, or low


molecular weight heparin (LMWH), given subcutaneously. Heparins generally act
by activating antithrombin III.

• Unfractionated heparin forms a complex which inhibits thrombin, factors Xa, IXa,
XIa and XIIa.

• LMWH however only increases the action of antithrombin III on factor Xa


What is the difference between heparin and
Low Molecular weight Heparin concerning
route, action duration, mechanism of action?
• Standard hepatin: Intravenous, Short, Activates antithrombin III. Forms acomplex
that inhibits thrombin, factors Xa, IXa, XIa and XIIa

• Low Molecular Weight: Subcutaneous, Long, Activates antithrombin III. Forms a


complex that inhibits factor Xa
What is the difference between heparin and
Low Molecular weight Heparin concerning
side effects and monitoring level?
• Standard Heparin: Bleeding, HIT, Osteoporosis, Activated partial thromboplastin
time (APTT)

• Low Molecular Weight Heparin: Bleeding, Lower risk of HIT and osteoporosis,
Anti-Factor Xa (although routine monitoring is not required)
What are the times in which we should use
standard heparin?
• Useful in situations where there is an increased risk of bleeding as
anticoagulation can be terminated rapidly
What are the times in which we should better
use low molecular weight heparin?
• Now standard in the management of venous thromboembolism
treatment and prophylaxis and acute coronary syndromes
What is Heparin-induced thrombocytopaenia
(HIT)?
• Immune mediated - antibodies form which cause the activation of platelets
• Usually does not develop until after 5-10 days of treatment
• Despite being associated with low platelets HIT is actually a prothrombotic
condition
• Features include a greater than 50% reduction in platelets, thrombosis and skin
allergy
• Treatment options include alternative anticoagulants such as lepirudin and
danaparoid
Why does Heparin cause Hyperkalemia?

• Both unfractionated and low-molecular weight heparin can cause


hyperkalaemia.

• This is thought to be caused by inhibition of aldosterone secretion.


What should you do in case of Heparin
overdose?
• Heparin overdose may be reversed by protamine sulphate, although
this only partially reverses the effect of LMWH.
What is Adrenaline?

• Adrenaline is a sympathomimetic amine with both alpha and beta adrenergic


stimulating properties

• Phentolamine, a short acting alpha blocker, may be used as local infiltration in


situations like accidental injection of adrenaline.

• It is normally used mainly to control blood pressure during surgical resection of


Pheochromocytoma.
What is the treatment of adrenaline induced
Ischemia?
• Adrenaline induced ischemia - phentolamine
What are the situations in which we use
Adrenaline?
• Anaphylaxis

• Cardiac arrest
What is the Recommend Adult Life
Support (ALS) adrenaline doses?
• Anaphylaxis: 0.5ml 1:1,000 IM

• Cardiac arrest: 10ml 1:10,000 IV or 1ml of 1:1000 IV


What is the Trough level?

• Trough level is the lowest level that a medicine is present in the body.

• In a medicine that is administered periodically, the trough level should be


measured just before the administration of the next dose in order to avoid
overdosing
When should the drug level be monitored in
case of Phenytoin and Cyclosporin?
• Phenytoin
• Trough levels immediately before dose

• Cyclosporin
• Trough levels immediately before dose
When should the drug level be monitored in
case of Digoxin and Lithium?
• Digoxin
• At least 6 hrs post-dose

• Lithium
• Range = 0.4 - 1.0 mmol/l
• Take 12 hrs post-dose
What are the uses of Bolulinum Toxin
(Botox)?
• Blepharospasm
• Hemifacial spasm

• Focal spasticity including cerebral palsy patients, hand and wrist disability
associated with stroke
• Spasmodic torticollis
• Severe hyperhidrosis of the axillae
• Achalasia
What is Isotretinoin?

• Isotretinoin is an oral retinoid used in the treatment of severe acne.

• Two-thirds of patients have a long term remission or cure following a


course of oral isotretinoin
What are the adverse effects of Isotretinoin?

• Teratogenicity: females MUST be using two forms of contraception (e.g. COCP and
condoms)
• Dry skin, eyes and lips: the most common side-effect of isotretinoin
• Low mood, depression
• Raised triglycerides
• Hair thinning
• Nose bleeds (caused by dryness of the nasal mucosa)
• Benign intracranial hypertension: isotretinoin treatment should not be combined with
tetracyclines for this reason
What are the SIGN guidelines on controlling
pain during palliative care?
• The breakthrough dose of morphine is one-sixth the daily dose of morphine

• All patients who receive opioids should be prescribed a laxative

• Opioids should be used with caution in patients with chronic kidney disease.
Alfentanil, buprenorphine and fentanyl are preferred

• metastatic bone pain may respond to NSAIDs, bisphosphonates or radiotherapy


What is the conversion table between
different types of opiod?
• Oral codeine to Oral morphine Divide by 10

• Oral tramadol to Oral morphine Divide by 5

• Oral morphine to Oral oxycodone Divide by 2


What is the equivalent of oral morphine in
patchs?
• The BNF states that oral morphine sulphate 80-90mg over 24 hours is
approximately equivalent to one '25 mcg/hour' patch, therefore product
literature should be consulted
What is the equivalent of oral morphine in
patchs?
• The BNF states that a fentanyl 75 patch is equivalent to 180mg daily intake of
morphine salt whilst a fentanyl 100patch is equivalent to 240mg daily morphine
salt intake.
Why do we use Fentanyl instead of morphine
for patients with Renal failure?
• Morphine is extensively metabolised in the liver to morphine-3-glucuronide
(M3G) and morphine-6- glucuronide (M6G), M6G is primarily excreted by the
kidneys, thus accumulation occurs in renal failure and is likely to cause opioid
toxicity
• Fentanyl is a synthetic opioid extensively metabolised by cytochrome P450
enzymes in the liver and gastrointestinal tract, thus it is unaffected by renal
impairment.
• Due to its extensive first pass metabolism, it can only be given parenterally (that
is, transmucosal, transdermal, intramuscular, intravenous or subcutaneous)
What is the conversion table between
different routes of opiod?
• Oral morphine to Subcutaneous diamorphine Divide by 3

• Oral oxycodone to Subcutaneous diamorphine Divide by 1.5


What is the Management of hiccups in
palliative care?

•Chlorpromazine is licensed for the treatment of intractable hiccups

•Haloperidol, gabapentin and baclofen are also used


How to treat agitation of a patient with
terminal illness?
• In the terminal phase of the illness (Care of the Dying pathway) then
agitation or restlessness is best treated with midazolam S/C
injection.
What is the Hydrocortisone Equivalence?

•1mg prednisolone = 4mg hydrocortisone

•1mg dexamethasone = 7mg prednisolone


What are the common side effects of
chemotherapy?
• Chemotherapy Side effects: Nausea and vomiting are common side-
effects of chemotherapy.
What are the Risk factors for the
development of symptoms include?
• Anxiety
• Age less than 50 years old

• Concurrent use of opioids

• The type of chemotherapy used


How to manage Nausea and vomiting caused
by chemotherapy?
• For patients at low-risk of symptoms then drugs such as metoclopramide may
be used first-line.

• For high-risk patients then 5HT3 receptor antagonists such as ondansetron are
often effective, especially if combined with dexamethasone
What is Medication overdose Headache?

• Medication overuse headache is one of the most common causes of


chronic daily headache. It may affect up to 1 in 50 people
What are the features of Medication overdose
headache?
• Present for 15 days or more per month

• Developed or worsened whilst taking regular symptomatic medication

• Patients using opioids and triptans are at most risk

• May be psychiatric co-morbidity


What is the management of medication
overdose headache?
• Simple analgesics and triptans should be withdrawn abruptly (may
initially worsen headaches)

• Opioid analgesics should be gradually withdrawn


• Give tricyclic antidepressants
What is Doxazosin?

• Doxazosin is an Alpha-1 adrenoceptor antagonist used in the


treatment of hypertension and benign prostatic hypertrophy
What are the Alpha Antagonists?

• Alpha-1: doxazosin
• Alpha-1a: tamsulosin - acts mainly on urogenital tract

• Alpha-2: yohimbine

• Non-selective: phenoxybenzamine (previously used in peripheral arterial


disease)
What are the Beta Antagonists?

•Beta-1: atenolol

•Non-selective: propranolol
Are Carvedilol and Labetalol Alpha or Beta
antagonists?
• Carvedilol and labetalol are mixed alpha and beta antagonists
What is Lithium?

• Lithium is mood stabilising drug used most commonly


prophylatically in bipolar disorder but also as an adjunct in refractory
depression.

• It has a very narrow therapeutic range (0.4-1.0 mmol/L) and a long


plasma half-life being excreted primarily by the kidneys
What kind of tremors come with Lithium
treatment?
• Lithium: fine tremor in chronic treatment,

• coarse tremor in acute toxicity


What is the difference between Lithium
overdose picture and Tricyclic?
• Tricyclic overdose may present with seizures but it does not typical
cause a tremor
What is the mechanism of action of Lithium?

• Mechanism of action - not fully understood, two theories:

• Interferes with inositol triphosphate formation

• Interferes with cAMP formation


What are the Adverse effects of Lithium?

• Nausea/vomiting, diarrhea

• Fine tremor
• Polyuria

• Thyroid enlargement, may lead to hypothyroidism


• ECG: T wave flattening/inversion
• Weight gain
What are the guidelines in monitoring lithium
levels in patient treated?
• Lithium blood level should ‘normally’ be checked every 3 months.
Levels should be taken 12 hours post-dose

• Thyroid and renal function should be checked every 6 months

• Patients should be issued with an information booklet, alert card and


record book
What is the blood level of lithium that may
cause toxicity?
• Lithium toxicity generally occurs following concentrations > 1.5
mmol/L.
What are the factors that may precipitate
Lithium toxicity?
• Toxicity may be precipitated by dehydration, renal failure, diuretics
(Especially bendroflumethiazide) or ACE inhibitors and ARBs

• Osmotic diuretics and carbonic anhydrase inhibitors like acetazolamide lead to


decreased lithium concentration and may hence lead to increased symptoms of
bipolar disorder.
Which drugs may cause increased
neurotoxicity of lithium?
• BNF advises that neurotoxicity may be when lithium is given with
diltiazem or verapamil but there is no significant interaction with
amlodipine.
Which liver test is a good indicator for
hepatotoxicity of lithium?

• Gamma GT is a more sensitive measure of the effects of lithium on


the liver than AST or ALT
What are the features of Lithium toxicity?

• Coarse tremor (a fine tremor is seen in therapeutic levels)

• Acute confusion

• Seizure

• Coma
What is the management of Lithium
Toxicity?
• Mild-moderate toxicity may respond to volume resuscitation with normal saline
(with or without furosemide if needed to induce diuresis)

• Hemodialysis may be needed in severe toxicity

• Sodium bicarbonate is sometimes used but there is limited evidence to support


this. By increasing the alkalinity of the urine it promotes lithium excretion.
What is the presentation of a patient with
Tricyclic overdose?
• Tricyclic Overdose is a common presentation to A&E departments.

• Early features relate to anticholinergic properties: dry mouth, dilated pupils,


agitation, sinus tachycardia, blurred vision.
What are the features of Tricyclic severe
dose?
• Features of severe poisoning include:
• Arrhythmias

• Seizures

• Metabolic acidosis
• Coma
What are the ECG features associated with
Tricyclic overdose?
• (Decrease the vagal inhibition on the heart):
• Sinus tachycardia
• they block sodium membrane channels with slowing of membrane depolarization,
thus having quinidine-like effects on the myocardium which causes:
• Widening of QRS
• Prolongation of QT interval
• Widening of QRS > 100ms is associated with risk of seizures whilst QRS > 160ms
is associated with ventricular arrhythmias
What is the management of Tricyclic
Toxicity?
• IV bicarbonate may decrease the risk of seizures and arrhythmias in severe toxicity
• In refractory cases, intravenous lipid emulsion may be given second line to bind to
free drug and reduce toxicity
• Arrhythmias: class I-a (e.g. Quinidine) and class I-c antiarrhythmics (e.g.
Flecainide) are contraindicated as they prolong depolarisation. Class III drugs such
as amiodarone should also be avoided as they prolong the QT interval.
• Response to lignocaine is variable and it should be emphasized that correction of
acidosis is the first line in management of tricyclic induced arrhythmias
• Dialysis is ineffective in removing tricyclics
What is the mechanism of Sodium
bicarbonate effect on TCA cardiotoxicity?

• Sodium bicarbonate attenuates TCA cardiotoxicity via several


mechanisms:
• Alkalinisation of blood to a pH of 7.45-7.55 uncouples TCA from
myocardial sodium channels; also, additional sodium increases
extracellular sodium concentration, thereby improving the gradient
across the channel.
Which should be given first to manage arrythmia
from Tricyclic toxicity Lignocaine or correction
of acidosis?
• Response to lignocaine is variable and it should be emphasized that
correction of acidosis is the first line in management of tricyclic
induced arrhythmias
Dialysis is helpful in treating tricyclic
toxicity, T/F?

• False

• Dialysis is ineffective in removing tricyclics


What are the acute side effects of Phenytoin?

• Initially: vertigo, diplopia, nystagmus, slurred speech, ataxia

• Later: confusion, seizures


What are the main features of Phenytoin
toxicity?
• Horizontal nystagmus is the sine qua non of phenytoin toxicity in this
group of patients.
• Nystagmus occurs with a concentration of at least 20 mg/l (80
mmol/l) and ataxia occurs at levels of 30–40 mg/l (120–160 mmol/l.
What are the Chronic side effects of
Phenytoin?
• Common: gingival hyperplasia, hirsuitism, coarsening of facial features
• Megaloblastic anemia (secondary to altered folate metabolism)
• Peripheral neuropathy

• Enhanced vitamin D metabolism causing osteomalacia


• Lymphadenopathy
• Dyskinesia
What are the Idiosyncratic (occur rarely and
unpredictably amongst the population) side effects of
Phenytoin?
• (TEN+Hepaitis +Aplastic Anemia)
• Fever
• Rashes, including severe reactions such as toxic epidermal necrolysis
• Hepatitis
• Dupuytren's contracture (although not listed in the BNF)
• Aplastic anemia
• Drug-induced lupus
What is the Teratogenic effect of Phenytoin?

•Associated with cleft palate and congenital heart disease


What is Sodium Valproate?

• Sodium Valproate is used in the management of epilepsy and is first


line therapy for generalised seizures.

• It works by increasing GABA activity


What are the side effects with sodium
valproate?
• Gastrointestinal: nausea
• appetite and weight gain
• Alopecia: regrowth may be curly (note that phenytoin hirsutism while valporate
alopecia)
• Ataxia
• Tremor
• Hepatitis (also with phenytoin)
• Pancreatitis , Teratogenic
What is the Anticholinesterase Effects?

• Anticholinesterase Effects: One of the effects of organophosphate


(for example, malathion and parathion) poisoning is inhibition of
acetylcholinesterase
What are the features of Anticholinesterase
Effects?
• Features can be predicted by the accumulation of acetylcholine (mnemonic =
SLUD)
• Salivation
• Lacrimation
• Urination
• Defecation
• Cardiovascular: hypotension, bradycardia
• Also: small pupils, muscle fasciculation
What is the management of organophosphate
poisoning (patient with anticholinestrase
activity)?
• Atropine + pralidoxime

• The role of pralidoxime is still unclear - meta-analyses to date have


failed to show any clear benefit
What is the difference between IV atropine
and Pralidoxime in treating organophosphate
poisoning?
• IV atropine may reverse the muscarinic effects but not the nicotinic
effects.
• Pralidoxime is a specific antidote that acts to regenerate the enzyme
activity at all affected sites.
What is St. John’s Wort?

• Shown to be as effective as tricyclic antidepressants in the treatment of mild-


moderate depression

• Mechanism: thought to be similar to SSRIS (although noradrenaline uptake


inhibition has also been demonstrated)
• NICE advise 'may be of benefit in mild or moderate depression, but its use
should not be prescribed or advised because of uncertainty about appropriate
doses, variation in the nature of preparations, and potential serious interactions
with other drugs'
What are the side effects of St. John’s Wort?

• Profile in trials similar to placebo

• Can cause serotonin syndrome

• Inducer of P450 system, therefore decrease levels of drugs such as


warfarin, Cyclosporin. The effectiveness of the combined oral
contraceptive pill may also be decreased
What are Monoamine Oxidase Inhibitors
(MAOIs)?

•Serotonin and noradrenaline are metabolised by monoamine oxidase


in the presynaptic cell
What are the Non-selective monoamine
oxidase inhibitors?
• E.g. tranylcypromine, phenelzine

• Used in the treatment of depression and other psychiatric disorder

• Not used frequently due to side-effects


What are the Adverse effects of non-
selective monoamine oxidase inhibitors?
• Hypertensive crisis: MAOIs reacting with tyramine containing foods e.g. Cheese,
pickled herring, Bovril, oxo, marmite, broad beans, liver, wine.

• Anticholinergic effects
What is the use of Serotonin Receptors
Medications?
• It should be noted that 5-HT receptor agonists are used in the acute
treatment of migraine whilst 5-HT receptor antagonists are used in
prophylaxis
What are the Seritonin receptors agonists?

• Sumatriptan is a 5-HT1D receptor agonist which is used in the acute


treatment of migraine

• Ergotamine is a partial agonist of 5-HT1 receptors


What are the serotonin receptors antagonists?

• Pizotifen is a 5-HT2 receptor antagonist used in the prophylaxis of migraine attacks.


• Methysergide is another antagonist of the 5-HT2 receptor but is rarely used due to the risk
of retroperitoneal fibrosis

• Cyproheptadine is a 5-HT2 receptor antagonist which is used to control diarrhea in patients with
carcinoid syndrome
• Olanzapine is 5-HT2 antagonist and D2 dopamin receptor blocker, it’s an atypical antipsychotic
• Ondansetron and Granisetron are 5-HT3 receptor antagonist and is used as an antiemetic...
They cause conistipation, dizziness and headache.
What are Triptans?

• Triptans: are specific 5-HT1 agonists used in the acute treatment of


migraine.

• They are generally used second line when standard analgesics such as
paracetamol and ibuprofen are ineffective
What are the guidelines when prescripting
Triptans?
• Should be taken as soon as possible after the onset of headache,
rather than at onset of aura

• Oral, orodispersible, nasal spray and subcutaneous injections are


available
What are the adverse effects of Tryptan?

• 'Triptan sensations' - tingling, heat, tightness (e.g. Throat and chest),


heaviness, pressure
What are the contraindications when giving
Triptans?
• Patients with a history of, or significant risk factors for ischemic heart
disease or cerebrovascular disease
What are the uses of Dopamine receptors
Agonists?
• Parkinson's disease

• Prolactinoma/galactorrhoea
• Cyclical breast disease

• Acromegaly
What are the adverse effects of Dopamine
receptors agonists?
• Nausea/vomiting

• Postural hypotension

• Hallucinations

• Daytime somnolence
What is the action of Benzodiazepines?

• Benzodiazepines enhance the effect of the inhibitory


neurotransmitter gamma-aminobutyric acid (GABA).
What are the uses of Benzodiazepines?

• They therefore are used for a variety of purposes (inhibit GABA):


• Sedation
• Hypnotic
• Anxiolytic

• Anticonvulsant

• Muscle relaxant
What is the problem of prescriping
benzodiazepines?
• Patients commonly develop a tolerance and dependance to benzodiazepines and
care should therefore be exercised on prescribing these drugs.

• The Committee on Safety of Medicines advises that benzodiazepines are only


prescribed for a short period of time (2-4 weeks).
How would you wean up a patient from
benzodiazepines?
• The dose should be withdrawn in steps of about 1/8 (range 1/10 to 1/4) of the
daily dose every fortnight. A suggested protocol for patients experiencing
difficulty is given:

• Switch patients to the equivalent dose of diazepam


• Reduce dose of diazepam every 2-3 weeks in steps of 2 or 2.5 mg

• Time needed for withdrawal can vary from 1 month to 1 year or more
What are the features of Benzodiazepines
withdrawal syndrome?
• If patients withdraw too quickly from benzodiazepines they may experience
benzodiazepine withdrawal syndrome, a condition very similar to alcohol
withdrawal syndrome.

• This may occur up to 3 weeks after stopping a long-acting drug.


What are the features of benzodiazepines
withdrawal syndrome?
• Insomnia, Irritability, Anxiety

• Tremor
• Loss of appetite
• Tinnitus
• Perspiration
• Perceptual disturbances
• Seizures
What are the 4 classes of antiarrythic drugs?

• "No Boy keeps clean."


Is a good mnemonic for Na, Beta, K, Ca blockers
What is Class I Antiarrythmic drugs?

• Class I (Na+ channel Blocker)


• IA (The Queen proclaims dis (this)  pyramid. )
• Increase AP (A is active)(Quinidine, Procinamide, Disopyramide, Amiodarone)
• IB (I Buy Lidy's Mexican tacos.)
• Decrease AP (B is lazy) (Lidocaine, Mexiletine, Tocainide, Phenytoin)

• IC (Can I have More Fries Please?) (C Conduction as it slows Conduction of AP)


• (Moricizine,Flecainide, Propafenone)
What is Class II Antiarrythmic drugs?

•Beta Blocker (Propanolol, Esmolol)


What is Class III Antiarrythmic drugs?

• ABDIS - A Big Dog Is Scary


• K+ channel Blockers ( Amiodarone, Bretylium, Dofetilide., Ibutilide,
Sotalol)
What is Class IV Antiarrythmic drugs?

• Ca++ Channel Blockers( Verapamil, Diltiazem)


What is Class V Antiarrythmic drugs?

• Adenosine

• Digoxin

• K+ ions

• Mg ions
What is Amiodarone?

• Amiodarone is a class III antiarrhythmic agent used in the treatment of both


atrial and ventricular tachycardias.

• The main mechanism of action is by blocking potassium channels which inhibits


repolarisation and hence prolongs the action potential.

• Amiodarone also has other actions such as blocking sodium channels (a class I-a
effect)
What are the limitations when using
Amiodarone?
• Long half-life (20-100 days)

• Should ideally be given into central veins (causes thrombophlebitis)

• Has proarrhythmic effects due to lengthening of the QT interval


• Interacts with drugs commonly used concurrently e.g. decreases metabolism of warfarin
= P450 inhibtor
• Numerous long-term adverse effects (see below)
What is the monitoring that you should do to
patients taking Amiodarone?
• TFT, LFT, U&E, CXR prior to treatment. U&E to check hypokalemia

• TFT, LFT every 6 months


What is the mechanism of causing most side
effects of Amiodarone?
• In affected patients, skin cells and the cells of other organs have been found to
contain myelin-like lysosomal structures and membrane-bound granules.
• This generalised derangement of lysosomal storage may also be the basis for
other adverse effects of amiodarone such as interstitial alveolitis, acute hepatitis,
and disturbed thyroid function.
What are the side effects of Amiodarone?

• Thyroid dysfunction
• Corneal deposits

• Pulmonary fibrosis/pneumonitis
• Liver fibrosis/hepatitis
• Peripheral neuropathy, myopathy
• Photosensitivity
• 'Slate-grey' appearance
What is the single most common side effects
of Amiodarone?
• The formation of corneal deposits is almost universal in patients taking
amiodarone therapy (at least 90%).
Around 1 in 6 patients taking amiodarone develop
thyroid dysfunction, T/F?

• TRUE
WHAT IS Amiodarone-induced
hypothyroidism (AIH)?
• The pathophysiology of amiodarone-induced hypothyroidism (AIH) is thought to
be due to the high iodine content of amiodarone causing a

• Wolff-Chaikoff effect (an autoregulatory phenomenon where thyroxine


formation is inhibited due to high levels of circulating iodide)
What is the difference between Amiodarone
Induced Thyrotoxicosis Type1 and Type2?

• Type1:
• Increased iodine,Increased thyroid hormone synthesis with present Goitre and
increased bloodflow in the coloured doplex , It’s managed by Carbimazole or
potassium perchlorate

• Type2:
• Amiodarone-related destructive thyroiditis, Goitre is Absent and decreased
bloodflow in Color Doppler, Management is by Corticosteroids ± Antithyroid
What is the difference between Amiodarone
Induced Hypothyroidism and Amiodarone
induced Thyrotoxicosis?
• Amiodarone can be continued if necessary even if Amiodarone induced
Hypothyroidism developed

• Unlike in AIH, amiodarone should be stopped if possible in patients who develop


AIT
What is Flecainide?

• Flecainide is a Vaughan Williams class I-c antiarrhythmic.

• It slows conduction of the action potential by acting as a potent sodium channel


blocker.

• This may be reflected by widening of the QRS complex and prolongation of the PR
interval
Can Flecainide be used to treat asymptomatic or mildly
symptomatic premature ventricular complexes?

• No
• Flecainide was actually shown to increase mortality post myocardial
infarction and is therefore contraindicated in this situation.
What are the indications for Flecainide?

• Atrial fibrillation

• Supra Ventricular Tachycardia associated with accessory pathway


e.g. Wolf-Parkinson-White syndrome
What are the adverse effects of Flecainide?

• Negatively inotropic
• Bradycardia
• Proarrhythmic

• Oral paraesthesia

• Visual disturbance
What is the action of Statins?

• Statins inhibit the action of HMG-CoA reductase, the rate-limiting enzyme in


hepatic cholesterol synthesis

• Statins decrease cholesterol synthesis.


Nicotinic acid increase HDL levels, T/F?

• True
Statins cause Myopathy, explain

• Statins cause Myopathy: includes myalgia, myositis, rhabdomyolysis and


asymptomatic raised creatine kinase.

• Risks factors for myopathy include advanced age, females, low BMI and
presence of multisystem disease such as diabetes mellitus.
• Myopathy is more common in lipophilic statins (simvastatin, atorvastatin)
than relatively hydrophilic statins (rosuvastatin, pravastatin, fluvastatin)
Statins cause liver impairment, how often
should you monitor liver function tests for a
patient that’s on Statins?
• 2008 NICE guidelines recommend checking LFTs at baseline, 3 months and 12
months.

• Treatment should be discontinued if serum transaminase concentrations rise to


and persist at 3 times the upper limit of the reference range
Who should take statins?

• All people with established cardiovascular disease (stroke, TIA, IHD, peripheral arterial disease)
• NICE recommend anyone with a 10-year cardiovascular risk = 20%
• The management of blood lipids in type 2 diabetes mellitus (T2DM) has changed slightly.

• Previously all patients with T2DM > 40-years-old were prescribed statins. Now patients > 40-
years-old who have no obvious cardiovascular risk (e.g. Non-smoker, not obese, normotensive etc)
and have a cardiovascular risk < 20%/10 years do not need to be given a statin.
What time of the day should you recommend
the patient to take the Statins dose?
• Statins should be taken at night as this is when the majority of cholesterol
synthesis takes place.

• This is especially true for simvastatin which has a shorter half-life than other
statins
What are the target levels for cholesterol
lowering according to different guidelines?
• Total cholesterol (mmol/l) LDL
cholesterol
• Joint British Societies < 4.0 < 2.0
• National Service Framework for CHD < 5.0 < 3.0
• SIGN 2007 < 5.0 < 3.0
current NICE guidelines do not recommend
target cholesterol in primary prevention, T/F?
• True
What are the main side effects of Statins
(HMG CoA reductase inhibitors) (used to
lower lipids)?
• Myositis, deranged LFTs
What are the main side effects of Ezetimibe
(drug used to lower lipids)?
• Headache
What are the main side effects of Nicotinic
acid (drug used to lower lipids)?
• Flushing, myositis

• Impaired Glucose Tolerance


What are the main side effects of Fibrates
(drug used to lower lipids)?
• Myositis, pruritus, cholestasis
What are the main side effects of Anion-
exchange resins (drug used to lower lipids)?
• GI side-effects
When you’re prescribing drugs for
hyperlipidemia, which combinations should
you avoid?
• Both fibrates and nicotinic acid have been associated with myositis, especially
when combined with a statin.

• However, the Committee on Safety of Medicines has produced guidance which


specifically warns about the concomitant prescription of fibrates with statins in
relation to muscle toxicity
What is Nicotinic Acid ?

• Nicotinic Acid is used in the treatment of patients with hyperlipidemia, although


its use is limited by side-effects.

• As well as lowering cholesterol and triglyceride concentrations it also raises HDL


levels
Beta-blocker overdose management:
atropine + glucagon, T/F?
• True
Why we use Glucagon in Beta blockers
toxicity?
• Glucagon has a positive inotropic action on the heart and decrease renal vascular
resistance.

• It is therefore useful in patients with Beta-blocker cardiotoxicity


• Immediate management is to give iv glucagon (50–150μg/kg) followed by a
glucagon infusion to treat hypotension and atropine if needed to treat
bradycardia. Because of the large quantities of glucagon required to achieve
therapeutic effect, a number of commentaries recommend insulin/dextrose
infusion as an alternative to glucagon.
When should you consider Cardiac pacing in
a patient with Beta Blocker Toxicity?
• Cardiac pacing should be reserved for patients unresponsive to
pharmacological therapy
What are the features of Beta-Blocker
Cardiotoxicity?

•Bradycardia

•Hypotension

•Heart failure

•Syncope
What is the management of Beta-Blocker
Cardiotoxicity?
• If bradycardiac then atropine

• In resistant cases glucagon may be used

• Hemodialysis is not effective in Beta-blocker overdose


What is Furosemide?

• Furosemide is a loop diuretic that acts by inhibiting chloride


absorption in the ascending loop of Henle.

• The name of Lasix is derived from lasts six (hours) referring to its
duration of action.
What are the side effects of Furosemide
(Hypo)?
• Hyponatremia, Hypokalemia, Hypocalcaemia
• Hypochloraemic alkalosis (Hyper pH)
• Ototoxicity
• Renal impairment (from dehydration + direct toxic effect)

• Hyperglycaemia (less common than thiazides)


• Gout
Which causes Hyperglycemia more often,
Furosemide or Thiazide?
• Thiazide
What is Bendroflumethiazide?

• Bendroflumethiazide (bendrofluazide) is a thiazide diuretic which works by


inhibiting sodium absorption at the beginning of the distal convoluted tubule
(DCT).

• Potassium is lost as a result of more sodium reaching the collecting ducts.

• Bendroflumethiazide has a role in the treatment of mild heart failure although


loop diuretics are better for reducing overload.
What is the use of Bendroflumethiazide?

• The main use of bendroflumethiazide currently is in hypertension


(part of the effect is due to vasodilation)
What are the common side effects of
Bendroflumethiazide?
• Dehydration
• Postural hypotension

• Hyponatremia, Hypokalemia, Hypercalcemia


• Gout
• Impaired glucose tolerance, Hyperglycaemia
• Impotence
What is the mechanism of
Bendroflumethiazide to cause Hypokalemia?
• Decrease sodium reaching the collecting ducts

• Activation of the renin-angiotensin-aldosterone


What are the rare side effects of
Bendroflumethiazide?
• Thrombocytopenia

• Agranulocytosis

• Photosensitivity rash

• Pancreatitis
What is Spironolactone?

• Spironolactone is an aldosterone antagonist which acts act in the


distal convoluted tubule
What are the indications of Spironolactone?

• Ascites: patients with cirrhosis develop a secondary hyperaldosteronism.


Relatively large doses such as 100 or 200mg are often used

• Heart failure (see RALES study below)


• Nephrotic syndrome
• Conn's syndrome
What are the side effects of spironolactone?

• Hyperkalemia

• Gynaecomastia
What is the effect of spironolactone in Heart
failure?
• NYHA III + IV, patients already taking ACE inhibitor

• Low dose spironolactone decrease all cause mortality


Which drugs increase or reduce the effect of
adenosine?
• The effects of adenosine are enhanced by dipyridamole (anti-platelet
agent) and blocked by theophyllines.

• It should be avoided in asthmatics due to possible bronchospasm.


What is the mechanism of action of
Adenosine?
• Causes transient heart block in the AV node

• Agonist of the A1 receptor which inhibits adenylyl cyclase thus reducing cAMP
and causing hyperpolarization by increasing outward potassium flux

• Adenosine has a very short half-life of about 8-10 seconds


What are the adverse effects of Adenosine?

• Chest pain

• Bronchospasm

• Can enhance conduction down accessory pathways, resulting in


increased ventricular rate (e.g. WPW)
Calcium channel blockers - side-effects: headache,
flushing, ankle edema, T/F?

• True
What are Calcium channel blockers?

• Calcium channel blockers are primarily used in the management of


cardiovascular disease.

• Voltage-gated calcium channels are present in myocardial cells, cells of the


conduction system and those of the vascular smooth muscle.

• The various types of calcium channel blockers have varying effects on these three
areas and it is therefore important to differentiate their uses and actions
What is the mood of action of Calcium
channel blockers?
• Decrease calcium entry to smooth and cardiac muscle which in turn
results in a decreased force of contraction and slower heart rate
What are the indications of using Calcium
channel blockers?
• Angina, hypertension, arrhythmias (e.g. Narrow complex
tachycardia), raynaud's
What do you know about Dihydropyridines
(e.g. nifedipine, amlodipine) (Calcium
channel blockers)?
• Effects peripheral circulation i.e. Used for hypertension, raynaud's

• May bring on angina due to sympathetic reflex following vasodilation

• Side-effects: headache, flushing, ankle edema


What do you know about Verapamil,
Diltiazem (Calcium channel blockers)?
• Contraindications: heart failure, heart block, on Beta-blockers

• Side-effects: headache, constipation, heart block


What are the indications for using Verapamil
?
• Angina, hypertension, arrhythmias

• Highly negatively inotropic


• Should not be given with beta-blockers as may
• cause heart block
What are the indications for using Diltiazam?

• Angina, hypertension

• Less negatively inotropic than verapamil but caution should still be


exercised when patients have heart failure or are taking beta-blockers
What are the indications for using Nifedipine,
amlodipine, felodipine (dihydropyridines)?

• Hypertension, angina, Raynaud's

• Affects the peripheral vascular smooth muscle more than the


myocardium and therefore do result in worsening of heart failure (but
not amlodepine)
What are the side effects/ contraindications of
Verapamil?
• Heart failure,
• constipation,
• hypotension,
• bradycardia
What are the side effects/ contraindications of
Diltiazam?
• Hypotension, bradycardia, heart failure, ankle swelling
What are the side effects/ contraindications of
Nifedipine, amlodipine, felodipine
(dihydropyridines)?
• Flushing, headache, ankle swelling
How does Aspirin work?

• Aspirin works by blocking the action of both cyclooxygenase-1 and 2.

• Cyclooxygenase is responsible for prostaglandin, prostacyclin and thromboxane


synthesis.

• The blocking of thromboxane A2 formation in platelets reduces the ability of


platelets to aggregate which has lead to the widespread use of low-dose aspirin
in cardiovascular disease.
Who should take Aspirin?

• All patients with established cardiovascular disease should take aspirin if there is
no contraindication.
Who should receive aspirin according to the
current guidelines?
• All people with established cardiovascular disease (stroke, TIA, IHD, peripheral
arterial disease)

• All people aged 50 years and over with a 10-year cardiovascular risk = 20%
• All people with diabetes mellitus (type 1 or 2) who are = 50 years old or who
have: diabetes > 10 years, taking treatment for hypertension or evidence of
target organ damage
• All people with target organ damage from hypertension
Which Drugs are potentiated by co-
administration of Aspirin?
• Oral hypoglycaemics

• Warfarin

• Steroids
What are Angiotensin-converting enzyme
(ACE) inhibitors ?
• Angiotensin-converting enzyme (ACE) inhibitors are now the established
first-line treatment in younger patients with hypertension
and are also extensively used to treat heart failure.

• They are known to be less effective in treating hypertensive Afro-Caribbean


patients.
• ACE inhibitors are also used to treat diabetic nephropathy and have a role in
secondary prevention of IHD.
What is the mechanism of action of
Angiotensin-converting enzyme (ACE)
inhibitors?
• Inhibit the conversion angiotensin I to angiotensin II
What are the side effects of Angiotensin-
converting enzyme (ACE) inhibitors?
• Cough: occurs in around 15% of patients and may occur up to a year after
starting treatment.

• Thought to be due to increased bradykinin levels


• Angioedema: may occur up to a year after starting treatment
• Hyperkalaemia
• 1st-dose hypotension: more common in patients taking diuretics
What are the Cautions and
contraindications of Angiotensin-
converting enzyme (ACE) inhibitors?
• Pregnancy and breastfeeding – avoid

• Renovascular disease - significant renal impairment may occur in patients who


have undiagnosed bilateral renal artery stenosis
• Aortic stenosis - may result in hypotension
• Patients receiving high-dose diuretic therapy (more than 80 mg of furosemide a
day) - signficantly increases the risk of hypotension
• Hereditary of idiopathic angioedema
What is the recommended monitoring for
patient taking Angiotensin-converting
enzyme (ACE) inhibitors?
• Urea and electrolytes should be checked before treatment is initiated and after
increasing dose

• A rise in the creatinine and potassium may be expected after starting ACE
inhibitors.

• Acceptable increases are an increase in serum creatinine, up to 50% from baseline


or up to 265 μmol/l (whichever is smaller) and an increase in potassium up to 5.5
mmol/l.
What are the drugs used with Acude Gout?

• Colchium alkaloids
• colchicine
• NSAIDS
• indoethacine
• Naproxen
• Phenylbutazone
• Ibuprofen
Colchicine interacts with Warfarin, T/F?

• Until recently it was thought that colchicine had little or no interaction with
warfarin therapy, but a case series was recently published that suggested INR is
increased in some patients prescribed colchicine.
• For patients on warfarin with acute attack of gout, rasburicase is recombinant
urate oxidase, which may be given during the attack
What are the features of Colchicine
toxicity ?
• It causes a myoneuropathy.
• This drug gives rise to a subacute proximal muscle weakness and on occasions
can lead to an acute necrotising myopathy.
• The muscle biopsy shows elements of both myopathic and neuropathic disease.
• The mechanism is not fully understood, but is attributable to probable
interference of the drug with tubulin, a protein required for the polymerisation
of microtubules in muscle and nerve.
• Weakness resolves when the drug is discontinued but the neuropathic features
remain.
What are the drugs that are used with Chronic
Gout?
• Uricosuric
• probenecid
• Sulfinpyazone

• Xanthine Oxidase Inhibitor


• allopurinol
What is Allopurinol?

• Allopurinol is used in the prevention of gout.

• It works by inhibiting xanthine oxidase which responsible for the


oxidation of 6-mercaptopurine to 6-thiouric acid
How to Initiate allopurinol prophylaxis?

• Allopurinol should not be started until 2 weeks after an acute attack has settled

• Initial dose of 100 mg od, with the dose titrated every few weeks to aim for a
serum uric acid of < 300 μmol/l

• NSAID or colchicine cover should be used when starting Allopurinol


What are the Indications for Allopurinol?

• Recurrent attacks - the British Society for Rheumatology recommend 'In


uncomplicated gout uric acid lowering drug therapy should be started if a second
attack, or further attacks occur within 1 year'
• Tophi
• Renal disease
• Uric acid renal stones
• Prophylaxis if on cytotoxics or diuretics
• Patients with Lesch-Nyhan syndrome often take allopurinol for life
What is the interaction between Allopurinol
and Azathioprine?
• Azathioprine is Metabolised to active compound 6- mercaptopurine

• Xanthine oxidase is responsible for the oxidation of 6- mercaptopurine to 6-


thiouric acid

• Allopurinol can therefore lead to high levels of 6-mercaptopurine


• A much decreased dose (e.g. 25%) must therefore be used if the combination
cannot be avoided
What is the interaction between Allopurinol
and Cyclophosphamide?
• Allopurinol decreases renal clearance, therefore may cause marrow
toxicity
What is Hormone Replacement Therapy?

• Hormone Replacement Therapy (HRT) involves the use of a small dose of


estrogen combined with a progestogen in (women with a uterus) to help alleviate
menopausal symptoms.

• The indications for HRT have changed significantly over the past ten years as the
long-term risks became apparent, primarily as a result of the Women’s Health
Initiative (WHI) study.
What are the Indications for using HRT
(Hormone Replacement therapy in post
menopausal women)?
• Vasomotor symptoms such as flushing, insomnia and headaches

• Premature menopause: should be continued until the age of 50 years

• Osteoporosis: but should only be used as second-line treatment


What is the main indication for using
Hormone Replacement Therapy?
• The main indication is the control of vasomotor symptoms.

• The other indications such as reversal of vaginal atrophy and prevention of


osteoporosis should be treated with other agents as first-line therapies.

• Other benefits include decrease incidence of colorectal cancer


What is the main side effects for using
Hormone Replacement Therapy?
• Nausea

• Breast tenderness

• Fluid retention and weight gain


What is the effect of Hormone Replacement
Therapy on Risks of different kinds of
Cancers?
• Increased Risk of breast cancer: increased by the addition of a progestogen

• Increased Risk of venous thromboembolism: increased by the addition of a


progestogen

• Decreased Risk of endometrial cancer: decreased by the addition of a


progestogen but not eliminated completely. The BNF states that the additional
risk is eliminated if a progestogen is given continuously
What is the relation between Hormone
Replacement Therapy (HRT/OCP) and the
risk of breast cancer?
• In the Women's Health Initiative (WHI) study there was a relative risk of 1.26 at 5
years of developing breast cancer
• The risk relates to duration of use
• Breast cancer incidence is higher in women using combined preparations
compared to estrogenonly preparations

• The risk of breast cancer begins to decline when HRT is stopped and by 5 years it
reaches the same level as in women who have never taken HRT
When to start a woman on Oral contraceptive
Pill?
• The decision of whether to start a woman on the combined oral
contraceptive pill is now guided by the UK Medical Eligibility Criteria
(UKMEC).
• UKMEC 1: a condition for which there is no restriction for the use of the
contraceptive method
• UKMEC 2: advantages generally outweigh the disadvantages
• UKMEC 3: disadvantages generally outweigh the advantages
• UKMEC 4: represents an unacceptable health risk
For Hormonal Replacement therapy for post
menopausal women, what are the features of UKMEC 3
women?
• More than 35 years old and smoking less than 15 cigarettes/day
• BMI 35-39 kg/m2
• Migraine without aura and more than 35 years old
• Family history of thromboembolic disease in first degree relatives < 45 years
• Controlled hypertension
• Breast feeding 6 weeks - 6 months postpartum
For Hormonal Replacement therapy for post
menopausal women, what are the features of UKMEC 4
women?
• More than 35 years old and smoking more than 15 cigarettes/day
• BMI > 40 kg/ m2
• Migraine with aura
• History of thromboembolic disease or thombogenic mutation
• History of stroke or ischemic heart disease
• Uncontrolled hypertension
• Breast cancer
• Major surgery with prolonged immobilisation
For Hormonal Replacement therapy for post
menopausal women under which classification does a
woman with Diabetes mellitus diagnosed > 20 years
ago fall?
• Diabetes mellitus diagnosed > 20 years ago is classified as UKMEC 3 or
4 depending on severity
What is Tamoxifen?

• Tamoxifen is a selective estrogen receptor modulator (SERM) which acts as an


estrogen receptor antagonist and partial agonist.

• It is used in the management of estrogen receptor positive breast cancer


What are the adverse effects of Tamoxifen?

• Hot flushes

• Menstrual disturbance: vaginal bleeding, amenorrhoea


• Venous thromboembolism
• Endometrial cancer
• Alopecia
• Cataracts
What is Raloxifen?

• Raloxifene is a pure estrogen receptor antagonist, and carries a lower


risk of endometrial cancer than Tamoxifen
What is Warfarin?

• Warfarin is an oral anticoagulant which inhibits the reduction of vitamin K to its


active hydroquinone form, which in turn acts as a cofactor in the formation of
clotting factor II, VII, IX and X (mnemonic = 1972) and protein C
INR also by antibiotics that kill intestinal flora
which will lead to decreased vitamin K
absorption, T/F?
• True
If you are a Dentist that’s going to have a
patient who is on Warfarin, what should you
do?
• Dentistry in warfarinised patients - check INR 72 hours before
procedure, proceed if INR < 4.0

• If patient has unstable INR then it should be checked 24H prior to


procedure
A patient that’s on warfarin who is going to
have a surgery, what should be done?

• In addition to discontinuing warfarin five days before surgery and


commencing low molecular weight heparin 48 hours later, INR
should be checked the day before the procedure to ensure that it is
below 1.5.
What are the factors that may potentiate the
effect of Warfarin?
• Liver disease
• P450 enzyme inhibitors, e.g.: amiodarone, ciprofloxacin
• Cranberry juice

• Drugs which displace warfarin from plasma albumin, e.g. NSAIDs


• Inhibit platelet function: NSAIDs
What are the side effects of Warfarin?

• Hemorrhage

• Teratogenic

• Skin necrosis:
Can you explain Skin necrosis that may be a
side effect of Warfarin?
• when warfarin is first started biosynthesis of protein C is decreased.

• This results in a temporary procoagulant state after initially starting warfarin,


normally avoided by concurrent heparin administration.

• Thrombosis may occur in venules leading to skin necrosis


What would you do for a patient who is
currently on warfarin and got Abnormally
high INR?
• If INR above 8, stop warfarin and:
• If there is a major bleeding give IV vit K 5mg + Fresh frozen plasma or
prothrombin concentrate
• If there is only Minor bleeding give IV vitamin K 1-3mg
• If there is no bleeding give oral Vitamin K 1-5mg
• If INR is between 5-8 , use the previous items for types of bleeding
but don’t give vitamin K if no bleeding (only stop the warfarin for two
doses and decrease the subsequent doses)
What should you do in case of patient on
Warfarin who has Major bleeding?
• Stop warfarin

• Vitamin K 5mg IV

• Prothrombin complex concentrate – (Preferred to FFP), if not available then FFP*


What should you do in case of patient on
Warfarin who has INR > 8.0 No bleeding or
minor bleeding?
• Stop warfarin, restart when INR < 5.0

• If risk factors for bleeding then give vitamin K 0.5mg IV or 5mg PO.

• Dose can be repeated after 24 hours if INR still high


What should you do in case of patient on
Warfarin who has INR 6.0 - 8.0 but No
bleeding or minor bleeding?
• Stop warfarin, restart when INR < 5.0
Which patient are at risk of bleeding if on
Warfarin and their INR got higher than 8.0?
• Risk factors include:
• Age > 70 years
• First year of warfarin therapy

• History of gastrointestinal bleeding


• Hypertension
• Alcohol excess
Why Prothrombin complex is favored over
FFP in case of Warfarin caused bleeding?
• FFP (Fresh Frozen Plasma) can take time to defrost, prothrombin
complex concentrate should be considered in cases of intracranial
hemorrhage
What do you know about Acyclovir
(Antiviral Drug)?
• Acyclovir is phosphorylated by thymidine kinase which in turn inhibits the viral
DNA polymerase
What do you know about Ribavirin
(Antiviral Drug)?
• Effective against a range of DNA and RNA viruses

• Interferes with the capping of viral mRNA


What do you know about Interferons
(Antiviral Drug)?
• Inhibit synthesis of mRNA, translation of viral proteins, viral assembly
and release
What do you know about Amantadine
(Antiviral Drug)?
• Used to treat influenza

• Inhibits uncoating of virus in cell


What do you know about Nucleoside Analogue Reverse
Transcriptase Inhibitors (NRTI) (Anti-retroviral agent
used in HIV)?

• Examples: zidovudine (AZT), didanosine, lamivudine, stavudine,


zalcitabine
What do you know about Protease inhibitors
(PI) (Anti-retroviral agent used in HIV)?
• Inhibits a protease needed to make virus able to survive outside the
cell

• Examples: indinavir, nelfinavir, ritonavir, saquinavir


What is Abacavir hypersensitivity?

• Abacavir can cause a severe and sometimes fatal hypersensitivity reaction. It is a


cell mediated delayed hypersensitivity reaction which occurs around 6 weeks
after starting treatment. Symptoms are of fever, rash, malaise and
gastrointestinal symptoms.
• Patients without HLA B*5701 are highly unlikely to develop hypersensitivity.
Those with the HLA B*5701 allele have a 50% risk and therefore in these
individuals abacavir should be avoided.
• It is therefore routine to test HLA B*5701 status on all newly diagnosed patients
with HIV.
What do you know about Non-Nucleoside Reverse
Transcriptase Inhibitors (NNRTI) (Anti-retroviral
agent used in HIV)?

• examples: nevirapine, efavirenz


Which anti HIV drug causes disturbing
dreams?

• Efaverinz is a non-nucleoside reverse transcriptase inhibitor (NNRTI) and is


frequently known to cause disturbing dreams and other cognitive disturbances in
50% of patients in the first month of treatment.
What do you know about HIV: Anti-
Retrovirals therapy?
• Highly active anti-retroviral therapy (HAART) involves a combination of at least
three drugs, typically two nucleoside reverse transcriptase inhibitors
(NRTI) and either a protease inhibitor (PI) or a non-nucleoside reverse
transcriptase inhibitor (NNRTI).

• This combination both decreases viral replication and also decreases the risk of
viral resistance emerging
What do you know about HIV: anti-
retrovirals - P450 interaction?
• nevirapine (NNRTI): induces P450

• protease inhibitors: inhibits P450


When should you start the Antiretroviral
therapy for HIV patient?
• Anti-retroviral therapy has previously been delayed until CD4 counts were below
200 * 106/l.

• This was largely due to the toxicity of drugs and fear of resistance developing.

• Recent guidelines now suggest starting treatment when counts drop below
350 * 106/l
What do you know about Nucleoside
analogue reverse transcriptase inhibitors
(NRTI)?
• Examples: zidovudine (azt), didanosine, lamivudine, stavudine, zalcitabine

• General NRTI side-effects: peripheral neuropathy

• Zidovudine: anemia, myopathy, black nails, fat redistribution and biochemical


changes consistent with the insulin resistant

• Didanosine: pancreatitis
What do you know about Nucleoside
analogue reverse transcriptase inhibitors
(NRTI) effect on cardiac muscle?
• Nucleoside reverse transcriptase inhibitor (NRTI) therapy is thought to reduce
vascular responsiveness to acetyl choline, and hence lead to endothelial
dysfunction.
• Mitochondrial dysfunction induced by HAART may also lead to decreased
myocardial contractility.
Which Nucleoside analogue reverse
transcriptase inhibitors (NRTI) is
associated with hypoblasic anemia?
• Zidovudine (AZT) has been found to inhibit haemoglobin synthesis and globin
gene transcription, and toxic metabolites of AZT have been shown to cause
cytotoxicity.
• In severe cases, patients on AZT have been known to become transfusion
dependent.
• Macrocytosis is common in patients on AZT
What do you know about Non-nucleoside
reverse transcriptase inhibitors (NNRTI)?
• Examples: nevirapine, efavirenz

• Side-effects: p450 enzyme interaction (nevirapine induces), rashes


What do you know about Protease inhibitors
(PI)?
• Examples: indinavir, nelfinavir, ritonavir, saquinavir

• Side-effects: diabetes, hyperlipidemia, buffalo hump, central obesity, p450


enzyme inhibition

• Indinavir: renal stones, asymptomatic hyperbilirubinemia

• Ritonavir: a potent inhibitor of the p450 system


What is Cyclosporin?

• Cyclosporin is an immunosuppressant which decrease clonal proliferation of T


cells by reducing IL-2 release.

• It acts by binding to cyclophilin forming a complex which inhibits calcineurin, a


phosphotase that activates various transcription factors in T cells
Cyclosporin + tacrolimus - MOA: inhibit
calcineurin thus decreasing IL-2, T/F?
• True
What are the side effects of Cyclosporin?

• Nephrotoxicity, Hepatotoxicity
• Fluid retention, Tremor
• Hypertension
• Hyperkalemia
• Hypertrichosis
• Hyperplasia of gum
• Impaired glucose tolerance, hyperglycemia.
What are the uses of Cyclosporin?

• Crohn's disease
• Rheumatoid arthritis
• Psoriasis (has a direct effect on keratinocytes as well as modulating T cell
function)
• Following organ transplantation

• Pure red cell aplasia


What is Tacrolimus?

• Tacrolimus is a macrolide antibiotic and is used as an immunosuppressant to


prevent transplant rejection.
• It has a very similar action to Cyclosporin; the action of tacrolimus differs in that
it binds to a protein called FKBP rather than cyclophilin

• Tacrolimus is more potent than Cyclosporin and hence the incidence of organ
rejection is less.
• However, nephrotoxicity and impaired glucose tolerance is more common
What is Azathioprine?

• Azathioprine is metabolised to the active compound mercaptopurine, purine


synthesis inhibitor, inhibiting the proliferation of cells, especially
leukocytes/lymphocytes.

• It is an effective drug used alone in certain autoimmune diseases, or in


combination with other immunosuppressants in organ transplantation.
What are the cautions that you should take
when prescribing Azathioprine?
• Caution should be exercised when it is used in conjunction with purine analogues
such as allopurinol, you may give only 25% of the usual dose of azathioprine.

• Thiopurine methyltransferase (TPMT) deficiency is present in about 1 in 200


people and predisposes to azathioprine related pancytopaenia.

• A thiopurine methyltransferase (TPMT) test may be needed to look for


individuals prone to azathioprine toxicity
What are the side effects of Azathioprine?

• Bone marrow depression

• Nausea/vomiting

• Pancreatitis
What is Methotrexate?

• Methotrexate is an antimetabolite which inhibits dihydrofolate reductase, an


enzyme essential for the synthesis of purines and pyrimidines
What are the indications for Methotrexate?

• Rheumatoid arthritis

• Psoriasis

• Acute lymphoblastic leukaemia


What are the adverse effects of Methotrexate?

• Mucositis

• Myelosuppression

• Pneumonitis

• Liver cirrhosis
What do you know about neutropenia
associated with Methotrexate?
• Methotrexate even when used weekly may be associated with neutropenia in 1
of each 58 patients.
• Methotrexate should be discontinued at least temporarily, and Granulocyte
colony-stimulating factor (GCSF) has been investigated as a therapy to allow
more rapid recovery of white count
What is the relation between Methotrexate
therapy and pregnancy?
• Men and women should avoid pregnancy for at least 3 months after
treatment has stopped
What are the signs and management of
Methotrexate toxicity?
• Overdoses produce symptoms of nausea, vomiting, stomatitis, skin
rashes and gastrointestinal bleeding.
• Calcium folinate (leucovorin), a derivative of tetrahydrofolate,
bypasses the folate step blocked by methotrexate and can be used in
the management of an overdose.
What are the guidelines you should consider
when prescriping Methotrexate?
• Methotrexate is a drug with a high potential for patient harm.
• Methotrexate is taken weekly, rather than daily
• FBC, U&E and LFTs need to be regularly monitored before starting treatment and repeated
weekly until therapy stabilised, thereafter patients should be monitored every 2-3 months'
• Folic acid 5mg once weekly should be coprescribed, taken more than 24 hours after methotrexate
dose
• The starting dose of methotrexate is 7.5 mg weekly
• Only one strength of methotrexate tablet should be prescribed (usually 2.5 mg)
• Avoid prescribing trimethoprim or cotrimoxazole concurrently - increases risk of marrowaplasia
What is the mode of action of
Mycophenolate mofetil?
• Mycophenolate mofetil inhibits inosine monophosphate dehydrogenase
What is the mode of action of Azathioprine ?

• Azathioprine metabolised to the active compound mercaptopurine


a purine analogue that inhibits DNA synthesis.

• purine synthesis inhibitor


What is the mode of action of Methotrexate?

• antimetabolite which inhibits dihydrofolate reductase


What are the uses of Rituximab?

• Non- Hodgkin's lymphoma

• Rheumatoid arthritis in refractory rheumatoid disease


• Used off-label to treat difficult cases of multiple sclerosis, SLE and autoimmune
anemias

• Pure red cell aplasia, ITP, Evans syndrome, vasculitis.


What are the side effects of Rituximab?

• Flu-like illness

• Decreased BP during fever

• Tumor side pain


What is Finasteride?

• Finasteride is an inhibitor of 5--reductase, an enzyme which


metabolises testosterone into dihydrotestosterone.

• Its indications are:


• Benign prostatic hyperplasia
• Male-pattern baldness
What are the side effects of Finasteride?

• Impotence
• Decreased libido
• Ejaculation disorders

• Gynaecomastia and breast tenderness


• Finasteride causes decreased levels of serum prostate specific antigen
What are Bisphosphonates?

• Bisphosphonates are analogues of pyrophosphate, a molecule which decreases


demineralisation in bone.
• They inhibit osteoclasts by reducing recruitment and promoting apoptosis

• Whilst the development of any new problem following the introduction of a new
drug warrants medical review it is particularly important to warn patients
starting bisphosphonates about symptoms which could suggest an esophageal
reaction, especially with alendronate
Bisphosphonates (alendronate) can cause a
variety of esophageal problems, T/F?
• True
What are the uses of Bisphosphonates?

• Prevention and treatment of osteoporosis

• Hypercalcemia

• Paget's disease

• Pain from bone metatases


What are the side effects of Bisphosphonates?

• Esophageal reactions: oesophagitis, esophageal ulcers (especially alendronate)


• Osteonecrosis of the jaw

• MHRA has warned about an increased risk of atypical stress fractures of the
proximal femoral shaft in patients taking alendronate
What are the guidelines when taking oral
Bisphosphonates?

• 'Tablets should be swallowed whole with plenty of water while sitting or


standing;

• to be given on an empty stomach at least 30 minutes before breakfast (or


another oral medication);

• Patient should stand or sit upright for at least 30 minutes after taking tablet'
What is Sildenafil?

• Sildenafil is a phosphodiesterase type V inhibitor used in the treatment of


impotence

• Nicorandil (vasodilatory drug used to treat angina) has a nitrate component as


well as being a potassium channel activator

• The BNF recommends avoiding Alpha-blockers for 4 hours after sildenafil


Viagra - contraindicated by nitrates and
nicorandil, T/F?
• True
What are the contraindications of Sildenafil?

• Patients taking nitrates and related drugs such as nicorandil

• Hypotension

• Recent stroke or myocardial infarction

• Non-arteritic anterior ischemic optic neuropathy


What are the side effects of Sildenafil?

• Visual disturbances e.g. Blue discoloration, non-arteritic anterior ischemic


neuropathy
• Nasal congestion

• Flushing

• Gastrointestinal side-effects
What are Octreotides?

• Long-acting analogue of somatostatin

• Somatostatin is released from D cells of pancreas and inhibits the release of


growth hormone
What are the uses of Octreotides?

• Acute treatment of variceal hemorrhage


• Acromegaly

• Carcinoid syndrome

• Prevent complications following pancreatic surgery


• VIPomas
What are the side effects of Octreotides?

• Gallstones (secondary to biliary stasis)


What is Theophylline?

• Theophylline, like caffeine, is one of the naturally occurring methylxanthines.


• The main use of theophyllines in clinical medicine is as a bronchodilator in the
management of asthma and COPD
• The exact mechanism of action has yet to be discovered. One theory suggests
theophyllines may be a non-specific inhibitor of phosphodiesterase resulting in
cAMP.
• Other proposed mechanisms include antagonism of adenosine and prostaglandin
inhibition
What are the features of Theophylline
poisoning?
• Acidosis, Hypokalemia
• Vomiting

• Tachycardia, arrhythmias
• Seizures
What is the management of Theophylline
poisoning?
• Management of overdose includes aggressive iv correction of hypokalaemia, iv
diazepam for overdose-induced seizures.
• Activated charcoal

• Charcoal hemoperfusion in case of a large dose is preferable to hemodialysis


• To treat Supra ventricular tachycardia resulting from poisoning, use Beta blockers
or Verapamil if Beta blockers are contraindicated (cases of asthma)
Theophyllines are phosphodiesterase inhibitors while
sildenafil is phosphodiesterase type V inhibitor, T/F?

• True
What is the mechanism of Alcohol
withdrawal?
• Chronic alcohol consumption enhances GABA mediated inhibition in the CNS
(similar to benzodiazepines) and inhibits NMDA-type glutamate receptors

• Alcohol withdrawal is thought to lead to the opposite ( decrease inhibitory GABA


and increase NMDA glutamate transmission)
What is the management of Alcohol
Withdrawal?
• Benzodiazepines

• Carbamazepine also effective in treatment of alcohol withdrawal

• Phenytoin is said not to be as effective in the treatment of alcohol


withdrawal seizures
What are Proton Pump Inhibitors (PPI)?

• Proton Pump Inhibitors (PPI) are a group of drugs which profoundly decrease
acid secretion in the stomach.

• They irreversibly block the hydrogen/potassium adenosine triphosphatase


enzyme system (the H+/K+ ATPase) of the gastric parietal cell.

• Examples include omeprazole and lansoprazole.


What are Aminosalicylates?

• Aminosalicylates: 5-aminosalicyclic acid (5-ASA) is released in the colon


and is not absorbed.

• It acts locally as an anti-inflammatory.

• The mechanism of action is not fully understood but 5-ASA may inhibit
prostaglandin synthesis
What is Sulphasalazine?

• A combination of sulphapyridine (a sulphonamide) and 5-ASA

• Many side-effects are due to the sulphapyridine moiety: rashes, oligospermia,


headache, Heinz body anemia

• Other side-effects are common to 5-ASA drugs (see mesalazine)


Give examples of common
Aminosalicylates?
• Sulphasalazine

• Mesalazine

• Olsalazine
What is Mesalazine?

• A delayed release form of 5-ASA

• Sulphapyridine side-effects seen in patients taking sulphasalazine are avoided

• Mesalazine is still however associated with side-effects such as GI upset,


diarrhea, headache, agranulocytosis, pancreatitis, interstitial nephritis
pancreatitis is 7 times more common in
patients taking mesalazine than sulfasalazine,
T/F?
• True
What is Olsalazine?

•Two molecules of 5-ASA linked by a diazo bond, which is broken by colonic


bacteria
What are the common diseases that may be
treated with Immunoglobulins?
• Primary and secondary immunodeficiency

• Idiopathic thrombocytopenic purpura (ITP)

• Myasthenia gravis
• Guillain-Barre syndrome
Give examples to conditions that may be
given Immunoglobulins?
• Kawasaki disease
• Toxic epidermal necrolysis (TEN)

• Pneumonitis induced by CMV following transplantation

• Low serum IgG levels following hematopoietic stem cell transplant for malignancy
• Dermatomyositis
• Chronic inflammatory demyelinating polyradiculopathy
What do you know about Immunoglobulin
therapy?
• Formed from large pool of donors (e.g. 5,000)

• IgG molecules with a subclass distribution similar to that of normal blood

• Half-life of 3 weeks
METROnidazole, Flagyl is named so because
it runs after flagellated organisms, T/F?
• True
What is the side effect of
Bendroflumethiazide?
• This patient has hyponatraemia and hypokalaemia. 
A patient with diabetic gastropathy is taking
erythromycin, why and what are the complications
of that?
• Erythromycin has been associated with prolonged QT interval and
torsades de pointes and is used in diabetic gastropathy, although its
benefits in the condition are not entirely understood.

• Prolonged QT is defined as greater than 0.45 s.


What are the electrolyte changes seen with
Thiazide diuretics?
• Thiazide diuretics are associated with increased calcium
concentrations as well as raised urate
In erythropoietin (EPO) abuse by sportsmen,
how long it can be detected after the last
dose?
• It can be detected in urine for a few weeks following the most recent
injection
You have a patient under the effect of a
cholinergic drug, please describe the effect?
• Muscarinic: vomiting, diarrhoea, abdominal cramping, bronchospasm, miosis,
bradycardia, excessive salivation and sweating. Severe diaphoresis can lead to
dehydration with systemic hypovolaemia, resulting in shock.

• Nicotinic: muscle fasciculations, tremor and weakness. Death is usually caused by


respiratory muscle paralysis.

• BP and pulse may be decreased due to muscarinic effects, or increased due to nicotinic
effects.
What should you do when you have a patient
bitten by a snake?
• The affected area should be immobilised below the level of the heart, to
minimise venous return.
• Any constricting clothing should be removed.
• The victim should be kept warm, but caffeine-containing drinks and alcohol
should be avoided.
• Ice should not be used (as this has been associated with increased necrotic
complications), and incisions/tourniquets should only be used by an experienced
physician. Zoos with exotic snakes normally store anti-venom, which is most
effective when given as early as possible. Coral
What is Phenoxybenzamine?

• Phenoxybenzamine is a non-competitive α-adrenoreceptor


antagonist, which cannot be displaced by endogenous α-receptor
ligands.
What percentage of hospital inpatients are
said to suffer from adverse drug reactions?
• It is said that 10–20% of hospital inpatients are said to suffer from
adverse drug reactions.
In Drug studies what is the purpose of Phase
1 studies?
• Phase 1 studies take place in very small numbers of healthy human
subjects and are designed to establish initial safety and toxicity data.
In Drug studies what is the purpose of Phase
2 studies?
• Phase 2 studies commonly take place in around 500 patients and are
designed to evaluate safety and toxicity and effective dose range and
monitor pharmacokinetic and pharmacodynamic effects
In Drug studies what is the purpose of Phase
3 and 4 studies?
• phase 3 studies that are designed predominantly to establish efficacy
in a wider patient population, and use much larger groups of patients
(usually between 1000 and 2000 in total).
• Unfortunately, due to the frequency of occurrence, unusual adverse
events are often only identified during phase 4 (post-marketing
surveillance studies) involving very large numbers of subjects.
What is the most likely neurological side-
effect from Streptomycin, Cochlear or
Vestibular damage?
• The most common neurological side-effect is vestibular damage
causing vertigo and vomiting.
• Cochlear damage is less frequent and results in deafness. Other side-
effects include rashes, angioneurotic oedema and nephrotoxicity.
What is Mitotane?

• Mitotane is an adrenal cortical cytotoxic agent and has a major role in


the treatment of adrenal cortical adenocarcinomas
What is cinchonism?

• It consists of symptoms of vomiting, vertigo, tinnitus, headache and


blurred vision
• This can be brought on by both acute and chronic treatment with
quinidine and quinine. It can also cause a prolonged QT interval on
the electrocardiogram.
• Cinchona is the name of the tree from which quinine can be
obtained
How do capsacin (extract of capsicum)
work?
• It depletes substance P in affected neurones and is particularly
effective for nerve pain.
What do you know about Barbiturate
overdose?
• The barbiturates have largely been replaced by the benzodiazepines. They are sill
used for induction anaesthesia and as anti-epileptics.
• Barbiturate overdose causes drowsiness, respiratory depression, impaired co-
ordination and coma.
• They are inducers of hepatic enzymes. As a drug they are acidic and elimination
through the kidneys is aided by forced alkaline diuresis with sodium
bicarbonate. Other treatments include charcoal and haemodialysis.
What is Gemfibrozil?

• Gemfibrozil is a lipid-lowering agent that stimulates lipoprotein lipase


and lowers triglyceride levels.
• It tends to increase biliary excretion of cholesterol and hence
predisposes to the formation of cholesterol gallstones.
What is the effect of tricyclic
antidepressants?
• These act as antimuscarinics (hence the papillary dilatation),
antihistamines, cause blockade of noradrenaline at the adrenergic
receptors, and have a ‘quinidine-like’ effect on the myocardium.
Which drugs may induce seizures?

• Many pharmaceuticals have the ability to induce seizures, and these


include tricyclic antidepressants, salicylate, antihistamines and
theophylline
• non-steroidal anti-inflammatories (NSAIDs) are safe but mefenamic
acid (ponstan) is associated with seizures
What are the cases in which multiple-dose
activated charcoal could be used?
• Activated charcoal is usually given once
• There are a few indications for the use of activated charcoal to be
given in a multiple dose regime, in which it may act as a form of
gastrointestinal dialysis.
• These are carbamazepine, dapsone, phenobarbitone, digoxin,
theophylline and quinine.
What do you know about Paraquat
poisoning?
• Paraquat is a bipyridium herbicide used as a weedkiller, and if ingested can be incredibly toxic, affecting
almost every organ system. Local gastrointestinal irritation occurs initially, followed by hepatocellular
dysfunction, renal failure, and chemical pneumonitis/acute respiratory distress syndrome. Perhaps the
most well-known consequence of its ingestion is pulmonary fibrosis. This is thought to occur due to
oxygen free radical damage, with initial destruction of type I and II alveolar cells and Clara cells, for
which paraquat has a predilection, with an energy-dependent uptake via the polyamine pathway.
Superoxide radicals are formed in the lungs in the presence of oxygen, which exacerbates lung damage,
and therefore its application should be restricted unless absolutely necessary. The qualitative urine test
for paraquat is performed by adding 1 ml of urine to a solution of sodium dithionate and sodium
hydroxide. If the solution turns blue, paraquat is present, a lighter green indicates diquat or low
paraquat concentrations. If the test is positive, one can then take a plasma paraquat concentration,
which will give an idea of prognosis and whether haemodialysis/haemoperfusion is warranted, although
in practice a very small number of patients are likely to benefit.
What is the management of Paraquat
poisoning?
• 15 ml/kg of Fuller’s Earth
• Fuller’s Earth is the preferred installant post paraquat ingestion at a dose of 15 ml/kg, if it is not
available, then 2 g/kg of charcoal is an alternative.
What is Drug-induced pseudolymphoma
syndrome?
• It consists of a number of erythematous indurated papules and nodules.
• Biopsy of the lesions reveals extensive T-lymphocyte infiltration.
• Causes include antiepileptics (phenytoin, carbamazepine, lamotrigine and a
number of other agents). A number of antibiotics, antiarrhythmics ,
antidepressants and antihistamines have also been implicated.
• Treatment involves discontinuation of all possible causative drugs. It is necessary
to monitor resolution of symptoms as malignant transformation of skin lesions
has been reported
What do you know about Non-haemolytic
transfusion reactions presenting with
Pyrexia?
• Non-haemolytic transfusion reactions characterised by pyrexia are usually due to
the presence of cytokines within the transfused unit of blood.
• Despite these type of reactions following a benign course, it is still important to
differentiate them from haemolytic transfusion reactions.
• As such the transfusion should be stopped and the patient treated with
paracetamol.
Which two antibiotics cause sensorineural
hearing loss when injected IV?
• it has been known for many years that both erythromycin and
vancomycin lead to sensorineural hearing loss.
• Again, hearing loss has been reported with use of the IV formulation
in particular, and is probably resulting from the cmax achieved with IV
dosing.
In the early period of aspirin overdose is it
acidosis or Alkalosis?

• The early period of aspirin overdose is associated with respiratory


alkalosis, as characterised by raised pH and hypocapnia.
What happenes to sodium nitroprusside when
it is exposed to sunlight?
• Anyone unused to administering sodium nitroprusside may not be
aware that it degrades in sunlight to form cyanide.
• If happened in hospital you have to administer Dicobalt edetate
300mg IV bolus up to 3 times
What is the relationship between sodium
nitroprusside and vitamin B12 deficiency?
• Cobalamin plays a role in cyanide metabolism and B12 levels tend to
fall as cyanide levels rise, thus its use is not recommended in B12
deficiency states.
How quickly does Sodium nitroprusside work
when you give it for hypertension?

• Sodium nitroprusside has a rapid hypotensive effect that begins to


appear within 30–60 s of commencing the infusion.
• Sodium nitroprusside is a potent short-acting vasodilator, which acts
via the nitric oxide pathway to increase cGMP within smooth muscle
cells.
Pregnant women who abuse alcohol can cause their babies to
have Fetal alcohol syndrome, what are the features?

• Fetal alcohol syndrome is associated with:


• (1) mental retardation;
• (2) growth retardation;
• (3) mid-facial abnormalities.

• it appears that deficits remain throughout adulthood, with individuals who


experience fetal alcohol syndrome much more likely to suffer from behavioural
difficulties, alcoholism and to be involved in criminal activity
Why drugs that obey zero-order kinetics
needs to be monitored?
• With drugs that obey zero-order kinetics, such as phenytoin and
ethanol, their enzyme metabolism is saturable, which means only a
certain amount of the drug can be eliminated over a period of time.
• This means that for relatively small increases in administered
drug, relatively large increases in plasma concentration may
occur, leading to potential drug toxicity.
What are the drugs that obey zero-order
kinetics?
• Mnemonic 'Peas & WHEATS
• Phenytoin, Phenylbutazone
• Warfarin, Heparin, Ethanol, Aspirin, Theophylline, Tolbutamide
Salicylates
What is the difference between Zero order
kinetics and First order Kinetics?
• Where drugs display first-order kinetics, the rate at which a drug is metabolised
and eliminated is generally proportional to the dose given, with an exponential
reduction in plasma concentration over time.
• Each half-life of a drug displaying first-order kinetics is associated with a 50%
reduction in plasma concentration, and drugs may be considered eliminated after
3–5 half-lives.
A patient taking warfarin, which juice is
going to interact, Cranberry or Grapefruit?

• Cranberry juice is specifically recognized to inhibit CYP2C9, which is


the main route for warfarin metabolism.
• Grapefruit juice is an inhibitor of CYP3A4, and as such the major
interactions for grapefruit juice are with agents such as simvastatin
and cyclosporin.
TNF blockers are contraindicated if the
patient has Multiple sclerosis or Pregnancy,
T/F?
• True
What are the different types of Volatile
substance abuse?
• Volatile substance abuse includes the intentional inhalation of organic
solvents, vapours, gasoline, aerosol propellants, glues etc.
• They are either ‘bagged’ (sprayed into a plastic bag and inhaled till
the subject passes out), or ‘huffed’ (sprayed on to a cloth and held to
the mouth).
What are the features of Glue
sniffing(Volatile substance abuse) ?
• Features include euphoria, blurring of vision, ataxia, a feeling of
omnipotence, delirium, convulsions, and status epilepticus.
What are the complications of Glue
sniffing(Volatile substance abuse) ?
• Glue sniffing leads to nervous system problems including optic
atrophy, encephalopathy, cerebellar degeneration and sensorimotor
polyneuropathy.
• Aplastic anaemia and hepatic and renal damage can occur.
• Renal toxicity includes proteinuria, distal renal tubular acidosis (type
1) and renal calculi.
What is the management of Iron over dose?

• Ingestion of large amounts of iron may lead to acute circulatory collapse


accompanied by hypotensio.
• IV desferrioxamine at the rate of 15 mg/kg/h is an initial treatment of choice,
and starting it as early as possible is important.
• If hypotension/ circulatory collapse is not present then it may be given IM at the
rate of 2 g every 6–12 h.
• Even after 5 hrs, gastric lavage is indicated with instillation of desferrioxamine
into the stomach to reduce further iron absorption. In severe cases of overdose
haemodialysis may be indicated and rarely exchange transfusion.
A hallmark of iron toxicity is a raised blood glucose;
only few drugs may cause this in overdose, T/F?

• True
• Hypoglycaemia occurs later in presentation due to liver failure
Patients alive 72 h after Iron overdose
ingestion usually survive, T/F?
• True
In children, as little as 1–2 g of iron may
prove fatal, T/F?
• True
What are the indications for giving
deferoxamine in Iron toxicity?
• Shock
• Altered mental status
• Persistent gastrointestinal symptoms
• Metabolic acidosis
• Pills visible on radiographs
• Serum iron level greater than 500 µg/dL or estimated dose greater than 60
mg/kg of elemental iron have been shown to be fatal doses*
Activated charcoal can be used to treat iron
toxicity, T/F?
• False
• Activated charcoal is not used as it is a very poor adsorber of
iron.
What is the management of a patient who got
a seizure after carbamazepine toxicity?
• A second seizures indicates IV lorazepam being the management of
choice
Beta blockers can cause some problems with
erectile function, T/F?
• True
• This is seen in non cardio selective Betal blockers (atenolol for
example)
• It’s not seen with cardioselective (bisoprolol)
What is Reversible posterior leukoencephalopathy?

• A posterior leucoencephalopathy which resembles hypertensive


encephalopathy may follow the use of several drugs like cyclophosphamide,
ciclosporin, tacrolimus, cisplatin, cytarabine, intravenous methotrexate etc.
• It presents with headache, vomiting, seizures and cortical blindness.
• It has been suggested that ciclosporin alters the blood–brain barrier and the fluid
overload and hypertension which accompanies the use of ciclosporin underlies
the radiological changes.
What are the features of Isoniazid toxicity?

• It causes refractory convulsions and metabolic acidosis with an increased anion


gap and coma
• Isoniazid is recognised to inhibit both conversion of pyridoxine to its active form,
and lactate dehydrogenase, which is thought to contribute to presentation with
metabolic acidosis.
• Pyridoxine is a specific antidote for isoniazid overdose. Convulsions should be
controlled by intravenous diazepam (0.1–0.2 mg/kg) and pyridoxine, which act
synergistically.
What are the features of Arsenic poisoning?

• Features of a sensorimotor polyneuropathy.


• Mees lines (hyperkeratosis of the palms and some transverse lines on the
nails)
• There is increased risk of cancers in such patients.
• Arsenic can be detected in the hair and nails for months following exposure.
• Dimercaptosuccinic acid (DMSA, succimer) and penicillamine have been used
successfully in arsenic poisoning.
What is Malignant hyperthermia associated
with anesthesia?
• patient develops progressively increasing muscular rigidity instead of muscular
relaxation, as halothane or succinylcholine is introduced. Temperature rises steadily
up to 42–43 °C. Respiratory and heart rates escalate. Arterial paCO2 increases and
creatine phosphokinase rises to extraordinary levels and spills into in the urine
• Certain congenital myopathies also predispose to malignant hyperthermia, so family
history of Anesthesia problems should be taken
• Treatment is to immediately switch from volatile anaesthetics at the first hint of
masseter spasm or rise of body temperature to alternative agents such as propofol.
Intravenous dantrolene can be given
All jelly fish stings are treated with acidic
solutions except?
• it has been shown that sting cells from the Portuguese man-of war discharge in
response to acetic acid and vinegar solutions should therefore be avoided when
this species is known to be the cause of the sting.
• A slurry of sodium bicarbonate has been shown to be of benefit in treating stings
from all UK species - and is therefore a better choice when it is uncertain which
species is responsible.
What do you know about the hypersensitivity
reactions to penicillins?
• IgE mediated reaction to a β-lactam based antibiotic.
• These reactions typically occur within minutes to hours of exposure, however, if
this is the first exposure it can present later in the course of the treatment.
• Such reactions may demonstrate cross-reactivity between classes of β-lactam
antibiotics (penicillins, cephalosporins and carbapenems).
• Rates of cross-reactivity between penicillins and cephalosporins are often quoted
at around 10% though likely to be somewhat lower.
• Cross-reactivity is less frequent with carbapenem class antibiotics such as
meropenem but still a risk.
What is the treatment of a weeverfish sting?

• The sting of the fish causes intense pain at the site of the wound.
• The toxin produced by the fish is heat-labile and is denatured at
temperatures above 40°C.
• Treatment of a weeverfish sting should include cleansing of the
wound and immersion in hot water (as hot as can be borne), ideally
around 45°C.
What is Rivaroxaban?

• Rivaroxaban is a highly selective direct factor Xa inhibitor.


• Inhibition of factor Xa interrupts the intrinsic and extrinsic pathway of the
coagulation cascade, inhibiting both thrombin formation and development of
thrombi.
• Rivaroxaban does not inhibit thrombin and that it has no effect on platelets has
been demonstrated.
• Indications for factor Xa inhibition have now been extended to chronic atrial
fibrillation
What is the temporary effect of trimethoprim
on the blood test for renal function?
• This drug competes with creatinine for excretion in the nephron and
as a result will push the serum creatinine value up.
• The effect can be profound particularly in patients with underlying
chronic kidney disease.
What is Co-enzyme Q10?

• Co-enzyme Q10 is similar to vitamin K and reduces warfarin's anticoagulant


effect (warfarin exerts its anticoagulant effect though inhibition of the synthesis
of vitamin K dependent clotting factors).
What is Reye’s syndrome?

• Reye’s syndrome is the rarely occurring triad of encephalopathy, fatty liver


degeneration and elevated transaminases during a post-infectious period.
• The condition is documented to occur predominantly in patients under the age of
18 years and may commonly follow influenza or varicella infection. Case fatality
rate is very high at between 25% and 50% of patients. The key factor in avoiding
the syndrome is the cessation of aspirin use in children.
• Unfortunately there is no specific antidote and treatment is supportive, a key
factor being recognition and treatment of raised intracranial pressure.
What are the features of magnesium sulphate
toxicity?
• Magnesium toxicity is more common when intravenous magnesium is given in
absence of hypomagnesaemia such as pre-eclampsia and is exacerbated in the
presence of renal failure
• Signs of hypermagnesaemia include (Nausea, Vasodilatation Hypotension
(myocardial depression + vasodilatation)
• and in severe case (Double vision, Drowsiness, Loss of deep tendon reflexes,
Respiratory depression, coma)
Which antihistamine is most likely to cause
sedation?
• All antihistamines are potentially sedating but the newer agents are
said to be less sedating than the older types such as
chlorpheniramine and cyproheptadine.
How can you calculate the electrolytes needs
for a patient who has diarrhea?
• An adult's daily requirement of sodium and potassium is approximately 1 mmol /
Kg / day each. In addition, electrolytes lost via excessive secretions must also be
accounted for as recommended in NICE guidance on intravenous fluid
prescription for adults in hospital
• One litre of diarrhoea contains approximately 120 mmol of sodium and 15
mmol of potassium. Therefore, the patient's maintenance and additional
electrolyte requirements can be calculated. 
What is Dapagliflozin ?

• Dapagliflozin is a renal SGLT-2 (sodium glucose transporter) inhibitor which has


only recently obtained UK and US licensing authority.
• It causes an increase in glucose excretion by the kidney to lower serum glucose
concentrations. SGLT-2 is a sodium/glucose co-transporter protein in the
nephrons proximal tubule which reabsorbs glucose from the renal filtrate.
• These drugs cause heavy urinary glucose loss and can cause recurrent urinary
infections and candidiasis as well as hypoglycaemia, crystalluria and renal failure
due to osmotic diuresis
What is HIV viral tropism and what is
Maraviroc?
• HIV virus binds to the CD4 cell via the CD4 receptor. This interaction also depend
on viral interaction with the CD4 co-receptor. There are two forms of CD4 co-
receptor CCR5 and CXCR4.
• The test for viral tropism determines which of these co-receptors the HIV virus
will bind to. Maraviroc blocks HIV binding to CCR5 receptor, and therefore is an
effective drug in a 'CCR5 tropic' virus. It however is not effective if the virus is
'CXCR4 tropic' or 'dual tropic'.
• It is routine to test HIV viral tropism on diagnosis, or on viralogical failure to
assess if maraviroc is a therapeutic option.
What do you know about Thallium
poisoning?
• Thallium poisoning is a rare cause of painful polyneuropathy, mood
change and alopecia.
• Skin changes such as palm and sole scaling, stomatitis, glossitis,
keratitis, eczema and classical alopecia and Mee's lines on the nails
typically occur within two weeks of continual exposure.
• Treatment is chelation therapy with oral Prussian Blue
Hippie crack (nitrous oxide) can cause
profound B12 deficiency, T/F?
• True
• Nitrous oxide use can precipitate severe vitamin B12 deficiency with
pronounced neurological and haematological signs, particularly in
susceptible individuals

You might also like